Sie sind auf Seite 1von 148

http://www.majortests.com/sat/grammar.

php

1. Illiteracy is an enormous problem,A it affectsB millions of people worldwide,C and is an


impediment toD social progress.  No errorE.

A.
B.
C.
D.
E.

2. The company president has takenA steps to ensure that sheB can handle the pressure and
anxiety associated withC the job, includingD joining a yoga class and enlisting the support of a
network of friends. No errorE.

A.
B.
C.
D.
E.

3. If you are sure thatA you are in the right,B you would notC mind an independent examination
ofD the case. No errorE.

A.
B.
C.
D.
E.

4. The union insistedA on an increase in theirB members’C starting pay, and threatened to call a
strike if the company refused toD meet the demand. No errorE.

A.
B.
C.
D.
E.

5. Television viewers claim thatA the number of scenes depictingB alcohol consumption haveC
increased dramatically overD the last decade. No errorE.

A.
B.
C.
D.
E.

6. Employees with lessA personal problems areB likelyC to be moreD productive. No errorE.

A.
B.
C.
D.
E.

7. The three richest men in America haveA assets worth more thanB the combined assetsC of the
sixty poorest countries ofD the world. No errorE.

A.
B.
C.
D.
E.

8. ShipwreckedA on a desert island, coconuts and otherB fruits formedC the basis of the sailor’sD
diet. No errorE.

A.
B.
C.
D.
E.

9. Fifty percent of the people alive today haveA never made a phone call, butB thirty percent stillC
have no electricity connections to theirD homes. No errorE.

A.
B.
C.
D.
E.

10. The rhododendron, whichA ornaments so manyB English gardens, isC not native toD Europe.
No errorE.

A.
B.
C.
D.
E.

11. The farmer should not have beenA so careless asB to leave the door of the house unbolted
whenC he had goneD to bed. No errorE.

A.
B.
C.
D.
E.

12. A censusA of the island revealedB a population of onlyC 10,000 peopleD. No errorE.

A.
B.
C.
D.
E.

13. The engineer, who is renowned for his ingenuityA, has designedB a very uniqueC cooling
system for our new plant inD Spain. No errorE.

A.
B.
C.
D.
E.

14. Shoes of thoseA kind areB bad for the feet;C low heels areD better. No errorE.

A.
B.
C.
D.
E.

15. My father saw how muchA Uncle Tom was enjoyingB his early retirement, andC so he decided
to do the sameD. No errorE.

A.
B.
C.
D.
E.
identification of sentence errors test results
Practice Test 01
Question Your Answer Correct Answer Result Explanation
1. E A Wrong Explain
2. E D Wrong Explain
3. E C Wrong Explain
4. D B Wrong Explain
5. D C Wrong Explain
6. A A Correct  
7. D E Wrong Explain
8. A A Correct  
9. A B Wrong Explain
10. E E Correct  
11. A D Wrong Explain
12. E D Wrong Explain
13. C C Correct  
14. B A Wrong Explain
15. C D Wrong Explain
SAT identification of sentence errors
explanation
Test 01
1. Illiteracy is an enormous problem,A it affectsB millions of people worldwide,C and is an
impediment toD social progress.  No errorE.

A.
B.
C.
D.
E.

Correct Answer: A

Explanation:

This is an example of a type of error known as comma splice. You cannot use a comma to link
two complete sentences: use a semicolon.

SAT identification of sentence errors


explanation
Test 01
2. The company president has takenA steps to ensure that sheB can handle the pressure and
anxiety associated withC the job, includingD joining a yoga class and enlisting the support of a
network of friends. No errorE.

A.
B.
C.
D.
E.

Correct Answer: D

Explanation:
Misplaced modifier. Including seems to refer to job when it should refer to steps.

SAT identification of sentence errors


explanation
Test 01
3. If you are sure thatA you are in the right,B you would notC mind an independent examination
ofD the case. No errorE.

A.
B.
C.
D.
E.

Correct Answer: C

Explanation:

After a present tense in the if clause, we need a future tense in the main clause. Would should be
changed to will.

SAT identification of sentence errors


explanation
Test 01
4. The union insistedA on an increase in theirB members’C starting pay, and threatened to call a
strike if the company refused toD meet the demand. No errorE.

A.
B.
C.
D.
E.

Correct Answer: B

Explanation:
Change their to its because the union is a singular noun.

SAT identification of sentence errors


explanation
Test 01
5. Television viewers claim thatA the number of scenes depictingB alcohol consumption haveC
increased dramatically overD the last decade. No errorE.

A.
B.
C.
D.
E.

Correct Answer: C

Explanation:

Change have to has because the number requires a singular verb.

SAT identification of sentence errors


explanation
Test 01
6. Employees with lessA personal problems areB likelyC to be moreD productive. No errorE.

A.
B.
C.
D.
E.

Correct Answer: A

Explanation:

Use fewer not less for countable items.


SAT identification of sentence errors
explanation
Test 01
7. The three richest men in America haveA assets worth more thanB the combined assetsC of the
sixty poorest countries ofD the world. No errorE.

A.
B.
C.
D.
E.

Correct Answer: E

Explanation:

No error.

8. ShipwreckedA on a desert island, coconuts and otherB fruits formedC the basis of the sailor’sD
diet. No errorE.

A.
B.
C.
D.
E.

Correct Answer: A

Explanation:

This is an example of a dangling modifier. It implies that coconuts were shipwrecked. To correct
the sentence you could change Shipwrecked to “When he was shipwrecked”.

SAT identification of sentence errors


explanation
Test 01
10. The rhododendron, whichA ornaments so manyB English gardens, isC not native toD Europe.
No errorE.

A.
B.
C.
D.
E.

Correct Answer: E

Explanation:

No error.

SAT identification of sentence errors


explanation
Test 01
9. Fifty percent of the people alive today haveA never made a phone call, butB thirty percent stillC
have no electricity connections to theirD homes. No errorE.

A.
B.
C.
D.
E.

Correct Answer: B

Explanation:

But is inappropriate here; change to and.

SAT identification of sentence errors


explanation
Test 01
11. The farmer should not have beenA so careless asB to leave the door of the house unbolted
whenC he had goneD to bed. No errorE.

A.
B.
C.
D.
E.

Correct Answer: D

Explanation:

No need for a past perfect tense here: use went.

SAT identification of sentence errors


explanation
Test 01
12. A censusA of the island revealedB a population of onlyC 10,000 peopleD. No errorE.

A.
B.
C.
D.
E.

Correct Answer: D

Explanation:

The word people is redundant.

SAT identification of sentence errors


explanation
Test 01
13. The engineer, who is renowned for his ingenuityA, has designedB a very uniqueC cooling
system for our new plant inD Spain. No errorE.

A.
B.
C.
D.
E.

Correct Answer: C

Explanation:

You cannot say very unique. [Just as you cannot say more superior.]

SAT identification of sentence errors


explanation
Test 01
14. Shoes of thoseA kind areB bad for the feet;C low heels areD better. No errorE.

A.
B.
C.
D.
E.

Correct Answer: A

Explanation:

Kind is singular so we should use this or that, but not these or those.

SAT identification of sentence errors


explanation
Test 01
15. My father saw how muchA Uncle Tom was enjoyingB his early retirement, andC so he decided
to do the sameD. No errorE.

A.
B.
C.
D.
E.

Correct Answer: D

Explanation:

The words “do the same” are ambiguous. Change to “take early retirement” or something
similar.

SAT identification of sentence errors


explanation
Test 01
15. My father saw how muchA Uncle Tom was enjoyingB his early retirement, andC so he decided
to do the sameD. No errorE.

A.
B.
C.
D.
E.

Correct Answer: D

Explanation:

The words “do the same” are ambiguous. Change to “take early retirement” or something
similar.
SAT Critical Reading
The critical reading sections on SAT are designed to test your ability to read and understand
written English of the level you need to make the most of a university course. There are two
types of questions:

 Sentence Completion
 Reading Comprehension

These questions require a level of vocabulary that should not be a problem if you have been in
the habit of reading good books throughout your high school career. However, if you are not a
good reader, or if English is your second language, you will have to work hard to raise your
vocabulary to the required level. You can start with learning our Wordlists that have been
divided into ten units so that you can spread this learning over ten weeks. If you have more time
available it would be a good idea to read some classic books.

Practice tests
The practice tests we have designed for you in each of the areas, contain vocabulary of the level
used by the SAT test makers. Keep a notebook handy while doing the tests and jot down any
unfamiliar words. Look them up in a dictionary and learn them before taking any further tests.
Why not try one of our free tests now:

 10 Sentence Completion Practice Tests


 14 Reading Comprehension Practice Tests
Practice SAT Sentence Completion
 Sentence Completion Practice Test 1
 Sentence Completion Practice Test 2
 Sentence Completion Practice Test 3
 Sentence Completion Practice Test 4
 Sentence Completion Practice Test 5
 Sentence Completion Practice Test 6
 Sentence Completion Practice Test 7
 Sentence Completion Practice Test 8
 Sentence Completion Practice Test 9
 Sentence Completion Practice Test 10

Directions
Each sentence has one or two blanks. Choose the answer choice that contains the word or words
that best complete the sentence.

Example:

There are some people who think that only the poor and less educated people use slang, but this
idea is _________.

(A) accurate

(B) popular

(C) erroneous

(D) widespread

(E) ineffectual

SAT sentence completion practice test 01


1. Today Wegener's theory is ____ ; however, he died an outsider treated with ____ by the
scientific establishment.
A. unsupported - approval
B. dismissed - contempt
C. accepted - approbation
D. unchallenged - disdain
E. unrivalled - reverence

2. The revolution in art has not lost its steam; it ____ on as fiercely as ever.

A. trudges
B. meanders
C. edges
D. ambles
E. rages

3. Each occupation has its own ____ ; bankers, lawyers and computer professionals, for example,
all use among themselves language which outsiders have difficulty following.

A. merits
B. disadvantages
C. rewards
D. jargon
E. problems

4. ____ by nature, Jones spoke very little even to his own family members.

A. garrulous
B. equivocal
C. taciturn
D. arrogant
E. gregarious

5. Biological clocks are of such ____ adaptive value to living organisms, that we would expect
most organisms to ____ them.
A. clear - avoid
B. meager - evolve
C. significant - eschew
D. obvious - possess
E. ambivalent - develop

6. The peasants were the least ____ of all people, bound by tradition and ____ by superstitions.

A. free - fettered
B. enfranchised - rejected
C. enthralled - tied
D. pinioned - limited
E. conventional - encumbered

7. Many people at that time believed that spices help preserve food; however, Hall found that
many marketed spices were ____ bacteria, moulds and yeasts.

A. devoid of
B. teeming with
C. improved by
D. destroyed by
E. active against

8. If there is nothing to absorb the energy of sound waves, they travel on ____ , but their
intensity ____ as they travel further from their source.

A. erratically - mitigates
B. eternally - alleviates
C. forever - increases
D. steadily - stabilizes
E. indefinitely - diminishes

9. The two artists differed markedly in their temperaments; Palmer was reserved and courteous,
Frazer ____ and boastful.
A. phlegmatic
B. choleric
C. constrained
D. tractable
E. stoic

10. The intellectual flexibility inherent in a multicultural nation has been ____ in classrooms
where emphasis on British-American literature has not reflected the cultural ____ of our country.

A. eradicated - unanimity
B. encouraged - aspirations
C. stifled - diversity
D. thwarted - uniformity
E. inculcated - divide

11. The conclusion of his argument, while ____ , is far from ____ .

A. stimulating - interesting
B. worthwhile - valueless
C. esoteric - obscure
D. germane - relevant
E. abstruse - incomprehensible

12. In the Middle Ages, the ____ of the great cathedrals did not enter into the architects' plans;
almost invariably a cathedral was positioned haphazardly in ____ surroundings.

A. situation - incongruous
B. location - apt
C. ambience - salubrious
D. durability - convenient
E. majesty - grandiose
SAT sentence completion explanation
Test 01
1. Today Wegener's theory is ____ ; however, he died an outsider treated with ____ by the
scientific establishment.

A. unsupported - approval
B. dismissed - contempt
C. accepted - approbation
D. unchallenged - disdain
E. unrivalled - reverence

Correct Answer: D

Explanation:

Clue: ‘however’ indicates a contradictory statement, and therefore we need opposite ideas in the
two halves of the sentence.
So, Wegener’s theory is unchallenged(everyone accepts it); however he was treated with disdain
(contempt).
Also the word ‘outsider’ in the sentence indicates that the second blank will have to be a
negative word.
(approbation = approval; reverence = respect)

2. The revolution in art has not lost its steam; it ____ on as fiercely as ever.

A. trudges
B. meanders
C. edges
D. ambles
E. rages

Correct Answer: E

Explanation:

Clue: ‘as ever’ indicates that things have not changed, so the two halves of the sentence need to
say similar things.
So, if the revolution has not lost its steam, it will be going on as strongly as before. Therefore
rages is the best word.
Also the word ‘fiercely’ indicates that we need a strong word.
(trudges = walks as though tired; meanders = wanders; ambles = walks aimlessly)
3. Each occupation has its own ____ ; bankers, lawyers and computer professionals, for example,
all use among themselves language which outsiders have difficulty following.

A. merits
B. disadvantages
C. rewards
D. jargon
E. problems

Correct Answer: D

Explanation:

Clue: The second half of the sentence is talking about language, and it is amplifying what has
been said before the semicolon. Therefore, the first blank needs to be about language and
‘jargon’ is specialized language used in a profession.

4. ____ by nature, Jones spoke very little even to his own family members.

A. garrulous
B. equivocal
C. taciturn
D. arrogant
E. gregarious

Correct Answer: C

Explanation:

Clue: He ‘spoke very little’ and so he uses few words.


Therefore, taciturn is the best word.
(garrulous = talkative; equivocal = ambiguous; gregarious = sociable)

5. Biological clocks are of such ____ adaptive value to living organisms, that we would expect
most organisms to ____ them.

A. clear - avoid
B. meager - evolve
C. significant - eschew
D. obvious - possess
E. ambivalent - develop

Correct Answer: D

Explanation:
Clue: The sentence structure suggests that if biological clocks are a great advantage, then most
organisms would possess them. [Or, if they are not an advantage, then organisms will not have
them.]
Therefore, the obvious value, leads us to expect that organisms possess them.
(meager = small, slight; eschew = shun, avoid; ambivalent = ambiguous)

6. The peasants were the least ____ of all people, bound by tradition and ____ by superstitions.

A. free - fettered
B. enfranchised - rejected
C. enthralled - tied
D. pinioned - limited
E. conventional - encumbered

Correct Answer: A

Explanation:

Clue: the part after the comma expands on what has been stated. Also note the word ‘least’,
which here means ‘not’.
Therefore, the peasants were least free, because they were fettered (restricted or bound) by
superstition.
(enfranchised = given voting rights; enthralled = fascinated; pinioned = tied down; encumbered
= burdened)

7. Many people at that time believed that spices help preserve food; however, Hall found that
many marketed spices were ____ bacteria, moulds and yeasts.

A. devoid of
B. teeming with
C. improved by
D. destroyed by
E. active against

Correct Answer: B

Explanation:

Clue: ‘However’ indicates that the first part of the sentence is contradicted by the second.
Therefore, if people once believed that spices preserved food, then the person mentioned found
that spices did not preserve food, and in fact the spices were teeming (full of) bacteria etc. that
could destroy the food.

8. If there is nothing to absorb the energy of sound waves, they travel on ____ , but their
intensity ____ as they travel further from their source.
A. erratically - mitigates
B. eternally - alleviates
C. forever - increases
D. steadily - stabilizes
E. indefinitely - diminishes

Correct Answer: E

Explanation:

Clue: ‘but’ indicates something contradictory.


So, if the waves are not absorbed they travel on indefinitely (apparently for ever), but the
intensity gets less (diminishes) as they travel away.
Remember, the sentence has to make the best sense, so saying that sound intensity increases as
the waves travel is counter to common sense.
(erratically = not constantly; alleviates = makes less severe)

9. The two artists differed markedly in their temperaments; Palmer was reserved and courteous,
Frazer ____ and boastful.

A. phlegmatic
B. choleric
C. constrained
D. tractable
E. stoic

Correct Answer: B

Explanation:

Clue: Since both ‘differ’ we need opposites.


So, since ‘reserved’ and courteous’ are both good words, we need a ‘bad’ word for the blank.
Choleric means easily angered and so would be a good opposite to restrained.
(phlegmatic = calm, imperturbable; constrained = restrained; tractable = easily lead, docile;
stoic = having fortitude)

10. The intellectual flexibility inherent in a multicultural nation has been ____ in classrooms
where emphasis on British-American literature has not reflected the cultural ____ of our country.

A. eradicated - unanimity
B. encouraged - aspirations
C. stifled - diversity
D. thwarted - uniformity
E. inculcated - divide

Correct Answer: C
Explanation:

Clue: Try to understand the overall meaning here. The emphasis on one type of literature
(British-American) has not reflected the differences (diversity) in our country, which is
multicultural. And so the flexibility in our culture has been reduced or suppressed (stifled).
(eradicated = wiped out; unanimity = consensus, agreement; aspirations = hopes; thwarted =
prevented, inculcate = instill)

11. The conclusion of his argument, while ____ , is far from ____ .

A. stimulating - interesting
B. worthwhile - valueless
C. esoteric - obscure
D. germane - relevant
E. abstruse - incomprehensible

Correct Answer: E

Explanation:

Clue: ‘far from’ indicates the need for an opposite.


So, while the conclusion is abstruse (obscure, difficult to comprehend), it is not totally
incomprehensible.
(esoteric = obscure; germane = relevant)

12. In the Middle Ages, the ____ of the great cathedrals did not enter into the architects' plans;
almost invariably a cathedral was positioned haphazardly in ____ surroundings.

A. situation - incongruous
B. location - apt
C. ambience - salubrious
D. durability - convenient
E. majesty - grandiose

Correct Answer: A

Explanation:

Clue: The semicolon indicates that the second part of the sentence expands on the first part.
So, the second part tells us we are talking about the position, or situation of a cathedral. And
since the first part tells us that architects did not pay attention to situation, the cathedral was
positioned randomly in odd (incongruous) surroundings.
(incongruous = not matching, odd; apt = suitable; ambience = atmosphere, environment;
grandiose = on a grand scale)
SAT sentence completion practice test 02
1. Unwilling to admit that they had been in error, the researchers tried to ____ their case with
more data obtained from dubious sources.

A. ascertain
B. buttress
C. refute
D. absolve
E. dispute

2. Archaeology is a poor profession; only ____ sums are available for excavating sites and even
more ____ amounts for preserving the excavations.

A. paltry - meager
B. miniscule - substantial
C. average - augmented
D. judicious - penurious
E. modest - generous

3. The student was extremely foolhardy; he had the ____ to question the senior professor's
judgment.

A. wisdom
B. temerity
C. interest
D. trepidation
E. condescension

4. The formerly ____ waters of the lake have been polluted so that the fish are no longer visible
from the surface.

A. muddy
B. tranquil
C. stagnant
D. pellucid
E. rancid

5. After the accident, the nerves to her arm were damaged and so the muscles ____ through
disuse.

A. atrophied
B. contracted
C. elongated
D. invigorated
E. dwindled

6. Some critics maintain that Tennyson's poetry is uneven, ranging from the ____ to the ____.

A. sublime - elevated
B. trite - inspired
C. vacuous - inane
D. succinct - laconic
E. sonorous - voluble

7. After grafting there is a ____ of lymphocytes in the lymph glands; the newly produced
lymphocytes then move in to attack the foreign tissue.

A. diminution
B. proliferation
C. obliteration
D. paucity
E. attraction

8. One ____ the new scheme is that it might actually ____ just those applicants that it was
intended to encourage.

A. highlight of - stimulate
B. feature of - attract
C. problem with - induce
D. attraction of - intimidate
E. drawback of - daunt

9. Corruption is ____ in our society; the integrity of even senior officials is ____ .

A. growing - unquestioned
B. endangered - disputed
C. pervasive - intact
D. rare - corrupted
E. rife - suspect

10. In their day to day decision making, many senior managers do not follow the rational model
favored by orthodox management experts, but rather rely on intuitive processes that often appear
____ and ____.

A. cerebral - considered
B. heretical - judgmental
C. conscientious - logical
D. irrational - iconoclastic
E. capricious - deliberate

11. His characteristically ____ views on examination methods at university level have aroused
____ in those who want to introduce innovative and flexible patterns of assessment.

A. hidebound - antagonism
B. moderate - anger
C. reactionary - admiration
D. rigid - support
E. accommodating - annoyance

12. Our grandfather was an entertaining ____; he used to ____ us with marvelous anecdotes that
we, in our childlike simplicity, accepted unquestioningly.

A. rascal - bore
B. orator - intimidate
C. raconteur - regale
D. curmudgeon - surprise
E. tyrant - stupefy

SAT sentence completion explanation


Test 02
1. Unwilling to admit that they had been in error, the researchers tried to ____ their case with
more data obtained from dubious sources.

A. ascertain
B. buttress
C. refute
D. absolve
E. dispute

Correct Answer: B

Explanation:

Clue: The researchers were unwilling to admit that they were wrong. Therefore they would try to
support (buttress) their arguments.
(ascertain = find out; refute = prove wrong; absolve = forgive)

2. Archaeology is a poor profession; only ____ sums are available for excavating sites and even
more ____ amounts for preserving the excavations.

A. paltry - meager
B. miniscule - substantial
C. average - augmented
D. judicious - penurious
E. modest - generous

Correct Answer: A

Explanation:

Clue: The part after the semicolon expands upon the first part of the sentence.
So, since the first part tells us that there is no money in archaeology, then there will only be
small (paltry) amounts for excavating. Also ‘even more’ indicates that another similar word is
required. Thus, meager also means small.
(miniscule = tiny; augmented = increased; judicious = wise, just; penurious = poor)
3. The student was extremely foolhardy; he had the ____ to question the senior professor's
judgment.

A. wisdom
B. temerity
C. interest
D. trepidation
E. condescension

Correct Answer: B

Explanation:

Clue: The semicolon indicates that the second part is closely related to the first.
So, since the student was foolhardy (reckless) he was doing something unwise. It would be
reckless to question the professor’s judgment, and so we can say that he had the temerity
(excessive nerve, or daring) to question him.
Note that temerity has a negative connotation.
(trepidation = fear and hesitation; condescension = arrogance)

4. The formerly ____ waters of the lake have been polluted so that the fish are no longer visible
from the surface.

A. muddy
B. tranquil
C. stagnant
D. pellucid
E. rancid

Correct Answer: D

Explanation:

Clue: The word ‘formerly’ indicates that once things were different.
So, since now the waters are polluted so that fish cannot be seen, then formerly they must have
been unpolluted and clear (pellucid).
(tranquil = peaceful; stagnant = not moving; pellucid = transparently clear; rancid = stale)

5. After the accident, the nerves to her arm were damaged and so the muscles ____ through
disuse.

A. atrophied
B. contracted
C. elongated
D. invigorated
E. dwindled
Correct Answer: A

Explanation:

Clue: The sentence states that the muscles were not used and so we would expect them to waste
away. Atrophied means wasted away, or withered.
Note that dwindled means grew less but could not be used for muscles.
(invigorated = energized)

6. Some critics maintain that Tennyson's poetry is uneven, ranging from the ____ to the ____.

A. sublime - elevated
B. trite - inspired
C. vacuous - inane
D. succinct - laconic
E. sonorous - voluble

Correct Answer: B

Explanation:

Clue: ‘Ranging from something to something’, indicates that extremes are required. Also we are
told that the poetry is uneven, also indicating that opposites are required.
Therefore, trite (unoriginal) is a negative word, and inspired is a positive word.
(sublime = magnificently inspiring; vacuous = meaningless = inane; succinct = short and to the
point; laconic = using few words; sonorous = full of sound ; voluble = talking a lot)

7. After grafting there is a ____ of lymphocytes in the lymph glands; the newly produced
lymphocytes then move in to attack the foreign tissue.

A. diminution
B. proliferation
C. obliteration
D. paucity
E. attraction

Correct Answer: B

Explanation:

Clue: The semicolon indicates that the second part of the sentence amplifies the first. Also the
second part states that the lymphocytes are ‘newly produced’. So, we infer that there is a
production (proliferation) of these cells in the glands.
(diminution = decrease; proliferation = growing and multiplying; obliteration = wiping out;
paucity = shortage)
8. One ____ the new scheme is that it might actually ____ just those applicants that it was
intended to encourage.

A. highlight of - stimulate
B. feature of - attract
C. problem with - induce
D. attraction of - intimidate
E. drawback of - daunt

Correct Answer: E

Explanation:

Clue: Try to understand the logic of the sentence. The new scheme might actually do something
to ‘put off’ (daunt), the applicants that it was intended to encourage. The word ‘intimidate’
would also have been suitable for the second blank, but its partner ‘attraction’ cannot be
right...we do not want a positive word for a negative feature.
(stimulate = encourage = induce; daunt = intimidate = put off)

9. Corruption is ____ in our society; the integrity of even senior officials is ____ .

A. growing - unquestioned
B. endangered - disputed
C. pervasive - intact
D. rare - corrupted
E. rife - suspect

Correct Answer: E

Explanation:

Clue: the semicolon suggests that the second part expands upon the first part. So, if corruption is
rife (common), then we will doubt the integrity of the officials. Their integrity will be suspect
(doubtful).
(pervasive = spreading everywhere; rife = common)

10. In their day to day decision making, many senior managers do not follow the rational model
favored by orthodox management experts, but rather rely on intuitive processes that often appear
____ and ____.

A. cerebral - considered
B. heretical - judgmental
C. conscientious - logical
D. irrational - iconoclastic
E. capricious - deliberate
Correct Answer: D

Explanation:

Clue : The ‘but rather’ construction indicates an opposite. Since they do not follow a rational
model, we infer that they must seem irrational. Also, since they do not follow the orthodox, they
must be unorthodox (iconoclastic).
Note that ‘and’ usually links words with similar values (both positive or both negative etc.)
(cerebral = concerned with thinking; heretical = opposed to orthodoxy = iconoclastic;
capricious = whimsical, fickle)

11. His characteristically ____ views on examination methods at university level have aroused
____ in those who want to introduce innovative and flexible patterns of assessment.

A. hidebound - antagonism
B. moderate - anger
C. reactionary - admiration
D. rigid - support
E. accommodating - annoyance

Correct Answer: A

Explanation:

Clue: Follow the logic. If his views are flexible the people who want flexible methods will
approve. But, if his views are rigid thee same people will oppose them.
Hence, the best fit comes with hidebound (rigid) and antagonism (hostility).
(reactionary = ultraconservative; accommodating = flexible).

12. Our grandfather was an entertaining ____; he used to ____ us with marvelous anecdotes that
we, in our childlike simplicity, accepted unquestioningly.

A. rascal - bore
B. orator - intimidate
C. raconteur - regale
D. curmudgeon - surprise
E. tyrant - stupefy

Correct Answer: C

Explanation:

Clue: Grandfather used to tell stories (anecdotes). He was also entertaining. So, the best answer
will be that he was a raconteur (teller of stories) who regaled (entertained) the children.
(orator = good speaker; curmudgeon = grumpy person; tyrant = harsh ruler)
SAT Reading Comprehension
Reading Comprehension (also known as Critical Reading) questions test your ability to
understand a passage and answer questions on the basis of what is stated and implied in the
passage. You need to read the passage first so that you can identify the main idea of the passage
and appreciate features such as the author's tone and attitude as well as the organization of the
passage. Scroll back to the relevant point in the text as you do each question.

Passages on the SAT vary in length from short paragraphs that take 3 minutes to read and answer
two questions, to ones that take 15 minutes to read and answer 13 questions. One section will
contain two related long passages. Mini tests 11 - 14 contain one paragraph reading
comprehensions. Be sure to read the directions and the time allowed at the beginning of each of
our mini tests.

There is no shortcut to improving your critical reading ability. Practice does help - but if you are
making too many errors on our mini tests, consult your teacher or just choose some good books
and get down to some serious reading.

Practice SAT Reading Comprehension


 Reading Comprehension practice test 1
 Reading Comprehension practice test 2
 Reading Comprehension practice test 3
 Reading Comprehension practice test 4
 Reading Comprehension practice test 5
 Reading Comprehension practice test 6
 Reading Comprehension practice test 7
 Reading Comprehension practice test 8
 Reading Comprehension practice test 9
 Reading Comprehension practice test 10
 Reading Comprehension practice test 11
 Reading Comprehension practice test 12
 Reading Comprehension practice test 13
 Reading Comprehension practice test 14

Directions
The reading passage is accompanied by a set of questions based on the passage and any
introductory material that is given. Answer the questions according to what is stated or implied
in the passage.
In the case of a double comprehension:
The two passages are accompanied by questions based on the content of each and the
relationship between them. Answer the questions on the basis of what is stated or implied in the
reading extracts and any introductory material.
The extract is taken from a book written sixty years ago by a British scientist in which he
considers the relationship between science and society.

    The pioneers of the teaching of science imagined that its


    introduction into education would remove the conventionality,
    artificiality, and backward-lookingness which were characteristic;
    of classical studies, but they were gravely disappointed. So, too, in
5   their time had the humanists thought that the study of the classical
    authors in the original would banish at once the dull pedantry and
    superstition of mediaeval scholasticism. The professional
    schoolmaster was a match for both of them, and has almost
    managed to make the understanding of chemical reactions as dull
10  and as dogmatic an affair as the reading of Virgil's Aeneid.

    The chief claim for the use of science in education is that it


    teaches a child something about the actual universe in which he is
    living, in making him acquainted with the results of scientific
15  discovery, and at the same time teaches him how to think logically
    and inductively by studying scientific method. A certain limited
    success has been reached in the first of these aims, but practically
    none at all in the second. Those privileged members of the
    community who have been through a secondary or public school
20  education may be expected to know something about the
    elementary physics and chemistry of a hundred years ago, but they
    probably know hardly more than any bright boy can pick up from
    an interest in wireless or scientific hobbies out of school hours.
    As to the learning of scientific method, the whole thing is palpably
25  a farce. Actually, for the convenience of teachers and the
    requirements of the examination system, it is necessary that the
    pupils not only do not learn scientific method but learn precisely
    the reverse, that is, to believe exactly what they are told and to
    reproduce it when asked, whether it seems nonsense to them or
30  not. The way in which educated people respond to such quackeries
    as spiritualism or astrology, not to say more dangerous ones such
    as racial theories or currency myths, shows that fifty years of
    education in the method of science in Britain or Germany has
    produced no visible effect whatever. The only way of learning the
35  method of science is the long and bitter way of personal
    experience, and, until the educational or social systems are altered
    to make this possible, the best we can expect is the production of a
    minority of people who are able to acquire some of the techniques
    of science and a still smaller minority who are able to use and
40  develop them.

Adapted from: The Social Function of Science, John D Bernal (1939)


1. The author implies that the 'professional schoolmaster' (line 7) has

A. no interest in teaching science

B. thwarted attempts to enliven education

C. aided true learning

D. supported the humanists

E. been a pioneer in both science and humanities.

2. The author’s attitude to secondary and public school education in the sciences is

A. ambivalent

B. neutral

C. supportive

D. satirical

E. contemptuous

3. The word ‘palpably’ (line 24) most nearly means

A. empirically

B. obviously

C. tentatively

D. markedly

E. ridiculously
4. The author blames all of the following for the failure to impart scientific method through the
education system except

A. poor teaching

B. examination methods

C. lack of direct experience

D. the social and education systems

E. lack of interest on the part of students

5. If the author were to study current education in science to see how things have changed since
he wrote the piece, he would probably be most interested in the answer to which of the following
questions?

A. Do students know more about the world about them?

B. Do students spend more time in laboratories?

C. Can students apply their knowledge logically?

D. Have textbooks improved?

E. Do they respect their teachers?

6. Astrology (line 31) is mentioned as an example of

A. a science that needs to be better understood

B. a belief which no educated people hold

C. something unsupportable to those who have absorbed the methods of science


D. the gravest danger to society

E. an acknowledged failure of science

7. All of the following can be inferred from the text except

A. at the time of writing, not all children received a secondary school education

B. the author finds chemical reactions interesting

C. science teaching has imparted some knowledge of facts to some children

D. the author believes that many teachers are authoritarian

E. it is relatively easy to learn scientific method.

SAT reading comprehension explanation


Test 01
1. The author implies that the 'professional schoolmaster' (line 7) has

A. no interest in teaching science


B. thwarted attempts to enliven education
C. aided true learning
D. supported the humanists
E. been a pioneer in both science and humanities.

Correct Answer: B

Explanation:

When we look back to line 7, we read, "The professional schoolmaster was a match for both of
them, and has almost managed to make the understanding of chemical reactions as dull and as
dogmatic an affair as the reading of Virgil's Aeneid."
This tells us that the schoolmaster has made learning dull. And so we eliminate answers C and E
which imply he has done something good.
But to be sure of the answer we should also read the previous sentences. We learn that, "The
pioneers of the teaching of science imagined that its introduction into education would remove
the conventionality, artificiality, and backward-lookingness which were characteristic of
classical studies......" This section tells us that other people tried to alter the nature of education,
but the "professional schoolmaster was a match for both of them". He therefore prevented
(thwarted) these attempts, and the answer is B.

2. The author’s attitude to secondary and public school education in the sciences is

A. ambivalent
B. neutral
C. supportive
D. satirical
E. contemptuous

Correct Answer: E

Explanation:

To find the attitude, try asking yourself whether the author is positive, negative or neutral to the
subject. Then look for the evidence. Here, it is obvious that he thinks that nothing very valuable
is learned in school about science and scientific method. He is therefore negative. Eliminate the
neutral (A and B) words, and the positive (C), and then decide between D and E. He seems to be
expressing contempt rather than mocking. And so E is the best choice.

3. The word ‘palpably’ (line 24) most nearly means

A. empirically
B. obviously
C. tentatively
D. markedly
E. ridiculously

Correct Answer: B

Explanation:

Go back to the text and find a word of your own to replace ‘palpably’ before you even look at the
choices. We read, "As to the learning of scientific method, the whole thing is palpably a farce."
Here, I could substitute ‘obviously’ or ‘clearly’. As it happens, one of the words is there in the
choices. (B). If it had not been there, there would have been something sufficiently similar to
make a choice.

4. The author blames all of the following for the failure to impart scientific method through the
education system except

A. poor teaching
B. examination methods
C. lack of direct experience
D. the social and education systems
E. lack of interest on the part of students

Correct Answer: E

Explanation:

Be careful on ‘except’ questions. You are looking for something the author does not do.
He does blame poor teaching, (lines 7-10), exams (line 26), social and education systems (line
36), lack of direct experience (lines 34-38), but he never blames the students. Hence answer E.

5. If the author were to study current education in science to see how things have changed since
he wrote the piece, he would probably be most interested in the answer to which of the following
questions?

A. Do students know more about the world about them?


B. Do students spend more time in laboratories?
C. Can students apply their knowledge logically?
D. Have textbooks improved?
E. Do they respect their teachers?

Correct Answer: C

Explanation:

This is an ‘inference’ question. We need to find out what the author’s main complaint is. This
concern of the author will tells us what he would like to see. From lines 11 to 18 in particular we
learn that he is especially interested in whether a student can apply his or her knowledge. So, we
conclude that answer C is best.

6. Astrology (line 31) is mentioned as an example of

A. a science that needs to be better understood


B. a belief which no educated people hold
C. something unsupportable to those who have absorbed the methods of science
D. the gravest danger to society
E. an acknowledged failure of science

Correct Answer: C

Explanation:

Astrology is mentioned as a ‘quackery’. Quackery is something that claims to be true but is


actually based on falsity. He implies that people are fooled by astrology, but he also implies that
there are other more ‘dangerous’ ideas. So we eliminate A, B and D. It is not likely that astrology
is a ‘failure of science’, but it is something that scientists would not approve of. Hence answer C.

7. All of the following can be inferred from the text except

A. at the time of writing, not all children received a secondary school education
B. the author finds chemical reactions interesting
C. science teaching has imparted some knowledge of facts to some children
D. the author believes that many teachers are authoritarian
E. it is relatively easy to learn scientific method.

Correct Answer: E

Explanation:

This is an ‘except’ question. Be careful! You are looking for something that cannot be inferred
from the text. We can find evidence that the author finds reactions interesting (line 9), and that
children have learnt some facts (beginning of the second paragraph), and that he thinks teachers
are strict (line 10 and part of paragraph 2). We can also infer from the use of the phrase
‘privileged members’ (line 18) that he believes that not all received secondary education. But we
find that he thinks it is hard to learn scientific method- ‘The only way of learning the method of
science is the long and bitter way of personal experience". And so we choose E.
SAT reading comprehension practice test 02
The passage is taken from a description of the life of certain Pacific Islanders written by a
pioneering sociologist.

    By the time a child is six or seven she has all the essential
    avoidances well enough by heart to be trusted with the care of a
    younger child. And she also develops a number of simple
    techniques. She learns to weave firm square balls from palm
5   leaves, to make pinwheels of palm leaves or frangipani blossoms,
    to climb a coconut tree by walking up the trunk on flexible little
    feet, to break open a coconut with one firm well-directed blow of
    a knife as long as she is tall, to play a number of group games
    and sing the songs which go with them, to tidy the house by
10  picking up the litter on the stony floor, to bring water from the
    sea, to spread out the copra to dry and to help gather it in when
    rain threatens, to go to a neighboring house and bring back a
    lighted faggot for the chief's pipe or the cook-house fire.

      But in the case of the little girls all these tasks are merely
15  supplementary to the main business of baby-tending. Very small
    boys also have some care of the younger children, but at eight or
    nine years of age they are usually relieved of it. Whatever rough
    edges have not been smoothed off by this responsibility for
    younger children are worn off by their contact with older boys.
20  For little boys are admitted to interesting and important activities
    only so long as their behavior is circumspect and helpful. Where
    small girls are brusquely pushed aside, small boys will be
    patiently tolerated and they become adept at making themselves
    useful. The four or five little boys who all wish to assist at the
25  important, business of helping a grown youth lasso reef eels,
    organize themselves into a highly efficient working team; one boy
    holds the bait, another holds an extra lasso, others poke
    eagerly about in holes in the reef looking for prey, while still
    another tucks the captured eels into his lavalava. The small girls,
30  burdened with heavy babies or the care of little staggerers who are
    too small to adventure on the reef, discouraged by the hostility
    of the small boys and the scorn of the older ones, have
    little opportunity for learning the more adventurous forms of work
    and play. So while the little boys first undergo the
35  chastening effects of baby-tending and then have many
    opportunities to learn effective cooperation under the supervision
    of older boys, the girls' education is less comprehensive. They
    have a high standard of individual responsibility, but the
    community provides them with no lessons in cooperation with one
40  another. This is particularly apparent in the activities of young
    people: the boys organize quickly; the girls waste hours in
    bickering, innocent of any technique for quick and efficient
    cooperation.

Adapted from: Coming of Age in Samoa, Margaret Mead (1928)

1. The primary purpose of the passage with reference to the society under discussion is to

A. explain some differences in the upbringing of girls and boys


B. criticize the deficiencies in the education of girls
C. give a comprehensive account of a day in the life of an average young girl
D. delineate the role of young girls
E. show that young girls are trained to be useful to adults

2. The word 'brusquely' (line 22) most nearly means

A. quickly
B. gently
C. nonchalantly
D. abruptly
E. callously

3. The list of techniques in paragraph one could best be described as

A. household duties
B. rudimentary physical skills
C. important responsibilities
D. useful social skills
E. monotonous tasks

4. It can be inferred that the 'high standard of individual responsibility' (line 38) is

A. developed mainly through child-care duties


B. only present in girls
C. taught to the girl before she is entrusted with babies
D. actually counterproductive
E. weakened as the girl grows older.

5. The expression 'innocent of' (line 42) is best taken to mean

A. not guilty of
B. unskilled in
C. unsuited for
D. uninvolved in
E. uninterested in

6. It can be inferred that in the community under discussion all of the following are important
except

A. domestic handicrafts
B. well-defined social structure
C. fishing skills
D. formal education
E. division of labor

7. Which of the following if true would weaken the author's contention about 'lessons in
cooperation' (line 39) ?

I Group games played by younger girls involve cooperation


II Girls can learn from watching boys cooperating
III Individual girls cooperate with their mothers in looking after babies

A. I only
B. II only
C. III only
D. I and II only
E. I, II and III

8. Which of the following is the best description of the author's technique in handling her
material?
A. Both description and interpretation of observations.
B. Presentation of facts without comment.
C. Description of evidence to support a theory.
D. Generalization from a particular viewpoint.
E. Close examination of preconceptions.

SAT reading comprehension explanation


Test 02
1. The primary purpose of the passage with reference to the society under discussion is to

A. explain some differences in the upbringing of girls and boys


B. criticize the deficiencies in the education of girls
C. give a comprehensive account of a day in the life of an average young girl
D. delineate the role of young girls
E. show that young girls are trained to be useful to adults

Correct Answer: A

Explanation:

Primary purpose questions require you to find the overview of the passage - what the author
intended to convey. It is like finding a title. In this case, we find that the author wanted to
describe some aspects of the upbringing of girls in a particular society and contrast that with the
way boys were brought up. Hence, answer A seems best. (To eliminate the others note that she is
describing rather than ‘criticizing’, that she is not giving an account of only one day, and that she
is saying something wider than just how they are trained to be useful. The D answer is too wide
ranging.)

2. The word 'brusquely' (line 22) most nearly means

A. quickly
B. gently
C. nonchalantly
D. abruptly
E. callously

Correct Answer: D

Explanation:
Go back to line 22 and find a word of your own to substitute. "Where small girls are brusquely
pushed aside, small boys will be patiently tolerated"...we find that we need something the
opposite of ‘patiently’. So the word ‘abruptly’ is closest to the idea of ‘impatiently’.

3. The list of techniques in paragraph one could best be described as

A. household duties
B. rudimentary physical skills
C. important responsibilities
D. useful social skills
E. monotonous tasks

Correct Answer: D

Explanation:

Look carefully at the list of techniques. They include, household chores, playing, and making
decorative items. They are not ‘monotonous’ and they are more than just ‘physical’ or
‘household’. Not all of them could be described as ‘important responsibilities’ and so we
eliminate, and choose D.

4. It can be inferred that the 'high standard of individual responsibility' (line 38) is

A. developed mainly through child-care duties


B. only present in girls
C. taught to the girl before she is entrusted with babies
D. actually counterproductive
E. weakened as the girl grows older.

Correct Answer: A

Explanation:

Always read some lines before and some lines after the line reference. In this case, reading
considerably before the line shows us that ‘responsibility’ is mainly associated with baby-
tending. The boys also learn this responsibility when they are young (line 15-17). The best
answer is therefore, A.

5. The expression 'innocent of' (line 42) is best taken to mean

A. not guilty of
B. unskilled in
C. unsuited for
D. uninvolved in
E. uninterested in
Correct Answer: B

Explanation:

Go back to the sentence and substitute your own word. " ...the girls waste hours in bickering,
innocent of any technique for quick and efficient cooperation." Here, ‘unaware of’, or ‘without
knowing’ would fit well. The best answer of the choices given is ‘unskilled in’.

6. It can be inferred that in the community under discussion all of the following are important
except

A. domestic handicrafts
B. well-defined social structure
C. fishing skills
D. formal education
E. division of labor

Correct Answer: D

Explanation:

Check each one carefully. In an ‘except’ question you are looking for something that is not right.
There is no mention of formal education (schooling), and so answer D is best.

7. Which of the following if true would weaken the author's contention about 'lessons in
cooperation' (line 39) ?

I Group games played by younger girls involve cooperation


II Girls can learn from watching boys cooperating
III Individual girls cooperate with their mothers in looking after babies

A. I only
B. II only
C. III only
D. I and II only
E. I, II and III

Correct Answer: D

Explanation:

First identify the ‘author’s contention’ (argument). She is saying, "the community provides them
(girls) with no lessons in cooperation with one another." To weaken that contention, we need to
show that they do get some opportunity to learn cooperation with one another. Point I shows that
they can learn, and so does II. III suggests that they cooperate with mothers but not necessarily
with each other. So, I and II correct means answer choice D.
8. Which of the following is the best description of the author's technique in handling her
material?

A. Both description and interpretation of observations.


B. Presentation of facts without comment.
C. Description of evidence to support a theory.
D. Generalization from a particular viewpoint.
E. Close examination of preconceptions.

Correct Answer: A

Explanation:

The author is describing, but she is also telling us what these observations imply. (The sentence
in lines 37-40 is an example of an interpretation.) Hence, answer A.
SAT reading comprehension practice test 03
The passage is taken from a biography of Florence Nightingale who is mainly remembered for
her heroic work as a nurse during the Crimean War.

    The name of Florence Nightingale lives in the memory of the


    world by virtue of the heroic adventure of the Crimea. Had she
    died - as she nearly did - upon her return to England, her
    reputation would hardly have been different; her legend would
5   have come down to us almost as we know it today - that gentle
    vision of female virtue which first took shape before the adoring
    eyes of the sick soldiers at Scutari. Yet, as a matter of fact, she
    lived for more than half a century after the Crimean War; and
    during the greater part of that long period all the energy and all the
10  devotion of her extraordinary nature were working at their
    highest pitch. What she accomplished in those years of unknown
    labor could, indeed, hardly have been more glorious than her
    Crimean triumphs; but it was certainly more important. The true
    history was far stranger even than the myth. In Miss Nightingale's
15  own eyes the adventure of the Crimea was a mere incident -
    scarcely more than a useful stepping-stone in her career. It was the
    fulcrum with which she hoped to move the world; but it was
    only the fulcrum. For more than a generation she was to sit in
    secret, working her lever: and her real life began at the very
20  moment when, in popular imagination, it had ended.

    She arrived in England in a shattered state of health. The


    hardships and the ceaseless efforts of the last two years had
    undermined her nervous system; her heart was affected; she
    suffered constantly from fainting-fits and terrible attacks of utter
25  physical prostration. The doctors declared that one thing alone
    would save her - a complete and prolonged rest. But that was also
    the one thing with which she would have nothing to do. She had
    never been in the habit of resting; why should she begin now?
    Now, when her opportunity had come at last; now, when the iron
30  was hot, and it was time to strike? No; she had work to do; and,
    come what might, she would do it. The doctors protested in vain;
    in vain her family lamented and entreated, in vain her friends
    pointed out to her the madness of such a course. Madness? Mad -
    possessed - perhaps she was. A frenzy had seized upon her. As
35  she lay upon her sofa, gasping, she devoured blue-books, dictated
    letters, and, in the intervals of her palpitations, cracked jokes. For
    months at a stretch she never left her bed. But she would not rest.
    At this rate, the doctors assured her, even if she did not die, she
    would become an invalid for life. She could not help that; there
40  was work to be done; and, as for rest, very likely she might rest ...
    when she had done it.

    Wherever she went, to London or in the country, in the hills


    of Derbyshire, or among the rhododendrons at Embley, she was
    haunted by a ghost. It was the specter of Scutari - the hideous
45  vision of the organization of a military hospital. She would lay that
    phantom, or she would perish. The whole system of the
    Army Medical Department, the education of the Medical Officer,
    the regulations of hospital procedure ... rest? How could she rest
    while these things were as they were, while, if the like necessity
50  were to arise again, the like results would follow? And, even in
    peace and at home, what was the sanitary condition of the Army?
    The mortality in the barracks, was, she found, nearly double the
    mortality in civil life. 'You might as well take 1, 100 men every
    year out upon Salisbury Plain and shoot them,' she said. After
55  inspecting the hospitals at Chatham, she smiled grimly. 'Yes, this
    is one more symptom of the system which, in the Crimea, put to
    death 16,000 men.' Scutari had given her knowledge; and it had
    given her power too: her enormous reputation was at her back -
    an incalculable force. Other work, other duties, might lie before
60  her; but the most urgent, the most obvious, of all was to look to
    the health of the Army.

Adapted from: Eminent Victorians, Lytton Strachey (1918)

1. According to the author, the work done during the last fifty years of Florence Nightingale's
life was, when compared with her work in the Crimea, all of the following except

A. less dramatic
B. less demanding
C. less well-known to the public
D. more important
E. more rewarding to Miss Nightingale herself.

2. The 'fulcrum' (line 17) refers to her

A. reputation
B. mental energy
C. physical energy
D. overseas contacts
E. commitment to a cause

3. Paragraph two paints a picture of a woman who is

A. an incapacitated invalid
B. mentally shattered
C. stubborn and querulous
D. physically weak but mentally indomitable
E. purposeful yet tiresome

4. The primary purpose of paragraph 3 is to

A. account for conditions in the army


B. show the need for hospital reform
C. explain Miss Nightingale's main concerns
D. argue that peacetime conditions were worse than wartime conditions
E. delineate Miss Nightingale's plan for reform

5. The series of questions in paragraphs 2 and 3 are

A. the author's attempt to show the thoughts running through Miss Nightingale's mind
B. Miss Nightingale questioning her own conscience
C. Miss Nightingale's response to an actual questioner
D. Responses to the doctors who advised rest
E. The author's device to highlight the reactions to Miss Nightingale's plans

6. The author's attitude to his material is

A. disinterested reporting of biographical details


B. over-inflation of a reputation
C. debunking a myth
D. uncritical presentation of facts
E. interpretation as well as narration
7. In her statement (lines 53-54) Miss Nightingale intended to

A. criticize the conditions in hospitals


B. highlight the unhealthy conditions under which ordinary soldiers were living
C. prove that conditions in the barracks were as bad as those in a military hospital
D. ridicule the dangers of army life
E. quote important statistics

1. According to the author, the work done during the last fifty years of Florence Nightingale's
life was, when compared with her work in the Crimea, all of the following except

A. less dramatic
B. less demanding
C. less well-known to the public
D. more important
E. more rewarding to Miss Nightingale herself.

Correct Answer: B

Explanation:

‘Except’ questions need careful checking. Here you are looking for something that cannot be
said of Florence Nightingale’s work in the last fifty years of her life. If you re-read from line 11
"What she accomplished in those years of unknown labor could, indeed, hardly have been more
glorious than her Crimean triumphs; but it was certainly more important..." you will find
evidence that her work was ‘important’, ‘less well-known’, ‘less dramatic’, and also ‘rewarding’
to her. But you will not find evidence that it was ‘less demanding’, in fact it was arduous, and put
a strain on her health. Therefore we choose answer B.

2. The 'fulcrum' (line 17) refers to her

A. reputation
B. mental energy
C. physical energy
D. overseas contacts
E. commitment to a cause

Correct Answer: A

Explanation:

Re-read lines 11-20. You will see that the ‘fulcrum’ was the ‘stepping stone’ she
was to use to advance her aims. This stepping-stone was the reputation she had earned in the
Crimea. Hence, answer A.
3. Paragraph two paints a picture of a woman who is

A. an incapacitated invalid
B. mentally shattered
C. stubborn and querulous
D. physically weak but mentally indomitable
E. purposeful yet tiresome

Correct Answer: D

Explanation:

Paragraph two reveals the poor state of health of Ms. Nightingale. (Her heart was affected; she
suffered attacks of utter physical prostration etc.). But it also shows that she never gave up and
could not be put off her work.(She would not rest; there was work to be done and she would do it
etc.). Hence she was physically weak but mentally indomitable. Answer D.

4. The primary purpose of paragraph 3 is to

A. account for conditions in the army


B. show the need for hospital reform
C. explain Miss Nightingale's main concerns
D. argue that peacetime conditions were worse than wartime conditions
E. delineate Miss Nightingale's plan for reform

Correct Answer: C

Explanation:

The primary purpose of paragraph 3 is to explain what Ms. Nightingale wanted to do and why.
Answer C. Note that answer D is too general - it refers to peacetime and wartime conditions but
does not state that it is for the army, and so is unacceptable. Answer B is also too general - we
are not concerned with hospitals in general, only the army.

5. The series of questions in paragraphs 2 and 3 are

A. the author's attempt to show the thoughts running through Miss Nightingale's mind
B. Miss Nightingale questioning her own conscience
C. Miss Nightingale's response to an actual questioner
D. Responses to the doctors who advised rest
E. The author's device to highlight the reactions to Miss Nightingale's plans

Correct Answer: A

Explanation:
The questions are a rhetorical device used by the author to try to give us a flavor of the thoughts
that preoccupied Ms. Nightingale. Answer A. (If you re-read, you will see that they cannot be
actual questions or responses.)

6. The author's attitude to his material is

A. disinterested reporting of biographical details


B. over-inflation of a reputation
C. debunking a myth
D. uncritical presentation of facts
E. interpretation as well as narration

Correct Answer: E

Explanation:

The author is highly involved in his subject. He tries to make the matter interesting, and tries to
state what Ms. Nightingale’s thoughts and reactions were. This is best covered by saying he
narrates and interprets. Answer E.

7. In her statement (lines 53-54) Miss Nightingale intended to

A. criticize the conditions in hospitals


B. highlight the unhealthy conditions under which ordinary soldiers were living
C. prove that conditions in the barracks were as bad as those in a military hospital
D. ridicule the dangers of army life
E. quote important statistics

Correct Answer: B

Explanation:

Ms. Nightingale was not quoting actual figures (eliminate E). She was also not concerned about
conditions in hospitals in general (eliminate A) - she was concerned with military hospitals, and
the conditions in the army in general. The last line tells us that her main concern was the ‘health
of the army’. So her main point is that ordinary solders were living in unsanitary conditions, and
answer B is best. She is not ridiculing the dangers she is pointing them out (eliminate D). She is
not proving anything (eliminate C).
SAT Grammar
There are 49 grammar questions on the SAT writing section. Grammar accounts for over two
thirds of the marks on this section. Questions are of 3 types:

 Identification of sentence errors


 Sentence correction (improving sentences)
 Editing in context (improving paragraphs)

Practice SAT Grammar


 Identify sentence errors test 1
 Identify sentence errors test 2
 Identify sentence errors test 3
 Identify sentence errors test 4
 Identify sentence errors test 5
 Identify sentence errors test 6

 Sentence correction test 1


 Sentence correction test 2
 Sentence correction test 3

 Editing in context test 1


 Editing in context test 2

Grammar Focus: 50 essential grammar rules is an ideal companion to the free practice grammar
tests available here on majortests.com.
It contains fifty rules that are essential knowledge for sentence correction, identification of
sentence errors and editing in context questions that you will encounter on the SAT.

SAT identification of sentence errors practice


test 02
1. We have no choice butA to appoint Mary:B she is the bestC of the two candidates, and there isD
no prospect of finding more applicants. No errorE.

A.
B.
C.
D.
E.

2. The reason I willA not be goingB to Mexico this year is becauseC I will use up all my travel
money attendingD an important meeting in Singapore. No errorE.

A.
B.
C.
D.
E.

3. If you wereA to work at least four hours a day on the project, we wouldB complete it in a
shorterC time, and with lessD problems. No errorE.

A.
B.
C.
D.
E.

4. The manager tried hard to effectA a change in company policy, butB the owner, who steadfastly
refused to compromiseC, overruledD him on every point. No errorE.
A.
B.
C.
D.
E.

5. The new library is undoubtedlyA well stocked and functional, butB no one can say that itsC
atmosphere is anything like the old oneD. No errorE.

A.
B.
C.
D.
E.

6. My uncle, whoA wasB on vacation, along with my two cousins and IC, wentD fishing down by
the river. No errorE.

A.
B.
C.
D.
E.

7. HopefullyA, we will beB able to complete the building beforeC the rainy season sets inD. No
errorE.

A.
B.
C.
D.
E.

8. You wouldA have to choose herB, if you are looking forC the best athlete to representD the
school. No errorE.
A.
B.
C.
D.
E.

9. All the trapped miners beganA to loseB hope, itC had been twenty four hours sinceD the tunnel
collapsed. No errorE.

A.
B.
C.
D.
E.

10. BecauseA they played byB the rules, the members of the team were givenC a standing ovation
even though itD did not win the match. No errorE.

A.
B.
C.
D.
E.

11. Her avariciousA relatives assembled at the lawyer'sB office to hear the reading ofC Jemima's
willD. No errorE.

A.
B.
C.
D.
E.

12. He was not merelyA expected to contribute funds to the project, butB to work as hard asC the
otherD patrons. No errorE.
A.
B.
C.
D.
E.

13. None of usA knowsB what the outcome of the battle betweenC the coordinator and usD will be.
No errorE.

A.
B.
C.
D.
E.

14. Neither of my brothers doA anything to make life better for ourB parents whoC are both
suffering fromD arthritis. No errorE.

A.
B.
C.
D.
E.

15. The teacher satA down besidesB the frightened child and tried to reassureC him that the
monster was merelyD imaginary. No errorE.

A.
B.
C.
D.
E.
SAT identification of sentence errors
explanation
Test 02
1. We have no choice butA to appoint Mary:B she is the bestC of the two candidates, and there isD
no prospect of finding more applicants. No errorE.

A.
B.
C.
D.
E.

Correct Answer: C

Explanation:

Since there are only two candidates we must use better not best.

2. The reason I willA not be goingB to Mexico this year is becauseC I will use up all my travel
money attendingD an important meeting in Singapore. No errorE.

A.
B.
C.
D.
E.

Correct Answer: C

Explanation:

The correct usage is to say, “the reason is that”, not “the reason is because”

3. If you wereA to work at least four hours a day on the project, we wouldB complete it in a
shorterC time, and with lessD problems. No errorE.

A.
B.
C.
D.
E.
Correct Answer: D

Explanation:

Change less to fewer.

4. The manager tried hard to effectA a change in company policy, butB the owner, who steadfastly
refused to compromiseC, overruledD him on every point. No errorE.

A.
B.
C.
D.
E.

Correct Answer: E

Explanation:

No error.

5. The new library is undoubtedlyA well stocked and functional, butB no one can say that itsC
atmosphere is anything like the old oneD. No errorE.

A.
B.
C.
D.
E.

Correct Answer: D

Explanation:

Incorrect comparison. Atmosphere must be compared to atmosphere. We could write, “... its
atmosphere is anything like that of the old one.”

6. My uncle, whoA wasB on vacation, along with my two cousins and IC, wentD fishing down by
the river. No errorE.

A.
B.
C.
D.
E.
Correct Answer: C

Explanation:

Change I to me (object of a preposition).

7. HopefullyA, we will beB able to complete the building beforeC the rainy season sets inD. No
errorE.

A.
B.
C.
D.
E.

Correct Answer: A

Explanation:

Change to we hope because hopefully is an adverb and yet it is not modifying a verb.

8. You wouldA have to choose herB, if you are looking forC the best athlete to representD the
school. No errorE.

A.
B.
C.
D.
E.

Correct Answer: A

Explanation:

Change would to will.

9. All the trapped miners beganA to loseB hope, itC had been twenty four hours sinceD the tunnel
collapsed. No errorE.

A.
B.
C.
D.
E.

Correct Answer: C
Explanation:

Change the comma to a semi-colon since you can't join two sentences with a comma.

10. BecauseA they played byB the rules, the members of the team were givenC a standing ovation
even though itD did not win the match. No errorE.

A.
B.
C.
D.
E.

Correct Answer: D

Explanation:

It has no antecedent. Change to they or the team.

11. Her avariciousA relatives assembled at the lawyer'sB office to hear the reading ofC Jemima's
willD. No errorE.

A.
B.
C.
D.
E.

Correct Answer: E

Explanation:

No error.

12. He was not merelyA expected to contribute funds to the project, butB to work as hard asC the
otherD patrons. No errorE.

A.
B.
C.
D.
E.

Correct Answer: A

Explanation:
To make the sentence construction parallel, put not merely after expected.

13. None of usA knowsB what the outcome of the battle betweenC the coordinator and usD will be.
No errorE.

A.
B.
C.
D.
E.

Correct Answer: E

Explanation:

No error.

14. Neither of my brothers doA anything to make life better for ourB parents whoC are both
suffering fromD arthritis. No errorE.

A.
B.
C.
D.
E.

Correct Answer: A

Explanation:

Neither is singular and so we should change do to does.

15. The teacher satA down besidesB the frightened child and tried to reassureC him that the
monster was merelyD imaginary. No errorE.

A.
B.
C.
D.
E.

Correct Answer: B

Explanation:

Change besides to beside.


SAT sentence correction practice test 01
1. Trying to keep her balance on the icy surface, the last competitor's ski-tip caught the pole and
somersaulted into the soft snow.

A. the last competitor's ski-tip caught the pole and somersaulted into the soft snow.
B. the ski-tip of the last competitor caught the pole and somersaulted in the soft snow.
C. the last competitor caught the pole with the tip of her ski, and somersaulted into the soft
snow.
D. the last competitor caught the pole with her ski-tip, which made her somersault into the
soft snow.
E. the last competitor somersaulted into the soft snow when the tip of her ski was caught by
the pole.

2. The temperature dropped suddenly last night, which will mean that the shoots emerging from
the soil will be killed by the frost.

A. which will mean that the shoots emerging from the soil will be killed by the frost.
B. which will mean that the frost will kill the shoots emerging from the soil.
C. and this will mean that the shoots emerging from the soil will be killed by the frost.
D. and the resulting frost will kill the shoots that are emerging from the soil.
E. and as a result, the shoots will be killed by the frost, emerging from the soil.

3. The impostor eluded detection for so long because she conducted herself as though she were a
licensed practitioner.

A. as though she were a licensed practitioner.


B. as though she was a licensed practitioner.
C. like she was a licensed practitioner.
D. like as if she was a licensed practitioner.
E. as if she was a practitioner with a license.

4. Being abandoned by our friends is the cause of great sorrow for us.

A. Being abandoned by our friends is the cause of great sorrow for us.
B. Our being abandoned by our friends is the cause of great sorrow.
C. Being abandoned by our friends, we feel great sorrow.
D. Abandoned by our friends, sorrow is the result.
E. We feel great sorrow when our friends abandon us.

5. Among the many reasons for his defeat in the election was his arrogant assumption that his
constituents were incapable of understanding economic conditions, and his unwarranted attack
on his chief opponent.

A. was his arrogant assumption that his constituents were incapable of understanding
economic conditions
B. were his arrogant assumption that his constituents were incapable of understanding
economic conditions
C. were his arrogant assumptions that his constituents were incapable of understanding
economical conditions
D. were his arrogant assumption that his constituents would be incapable of understanding
economics
E. was the arrogant assumption that his constituents was incapable of understanding
economic conditions

6. More and more holidaymakers are choosing to fly to remote islands in search of the perfect
beach; seeking sand, sun and palm trees, rather than centers of entertainment.

A. ; seeking sand, sun and palm trees, rather than centers of entertainment.
B. ; seeking sad, sun, palm trees and not entertainment.
C. , with sand, sun, palm trees and no entertainment.
D. , they seek sand, sun and palm trees, rather than entertainment centers.
E. ; they seek sand, sun and palm trees, rather than centers of entertainment.

7. The government requires that these forms should be submitted before the end of the financial
year.

A. that these forms should be submitted


B. that these forms be submitted
C. for these forms to be submitted
D. these forms submission
E. these forms should be submitted
8. After arduous months of fighting, the sight of the white flag being raised generated as much
relief on the victor's side than it did on the vanquished.

A. as much relief on the victor's side than it did on the vanquished.


B. as much relief among the victors as among the vanquished.
C. as much relief on the victor's side as it did on the vanquished's.
D. relief both on the victor's side as well as on the vanquished's.
E. relief both for the victor and the vanquished side.

9. The best way to encourage innovative thinking is not to promise financial rewards for ideas,
but to ensure that the person making the suggestion receives recognition for his contribution.

A. but to ensure that the person making the suggestion receives recognition for his
contribution.
B. but to ensure that the person who makes the suggestion will be receiving recognition for
his contribution.
C. but rather by ensuring that the person making the suggestion receives recognition for his
contribution.
D. but rather ensure that suggestion-maker receives recognition for his contribution.
E. but instead make sure that the suggestion-maker will receive recognition.

10. It ought to be her with whom you share your secrets, not me.

A. her with whom you share your secrets, not me


B. her with whom you share your secrets, not I.
C. she with whom you share your secrets, not me.
D. she with whom you share your secrets, not I.
E. her with who you share your secrets, not me.

SAT sentence correction explanation


Test 01
1. Trying to keep her balance on the icy surface, the last competitor's ski-tip caught the pole and
somersaulted into the soft snow.
A. the last competitor's ski-tip caught the pole and somersaulted into the soft snow.
B. the ski-tip of the last competitor caught the pole and somersaulted in the soft snow.
C. the last competitor caught the pole with the tip of her ski, and somersaulted into the soft snow.

D. the last competitor caught the pole with her ski-tip, which made her somersault into the soft
snow.
E. the last competitor somersaulted into the soft snow when the tip of her ski was caught by the
pole.

Correct Answer: C

Explanation:

After the comma we need the name of the person. In answer D, the use of which is incorrect.
Answer E unnecessarily uses a passive construction.

2. The temperature dropped suddenly last night, which will mean that the shoots emerging from
the soil will be killed by the frost.

A. which will mean that the shoots emerging from the soil will be killed by the frost.
B. which will mean that the frost will kill the shoots emerging from the soil.
C. and this will mean that the shoots emerging from the soil will be killed by the frost.
D. and the resulting frost will kill the shoots that are emerging from the soil.
E. and as a result, the shoots will be killed by the frost, emerging from the soil.

Correct Answer: D

Explanation:

The word which does not have an antecedent noun, and so the construction is incorrect in A and
B. Similarly, in C this does not have a noun to which it refers. D is the best choice. In E the
phrase emerging from the soil is dangling.

4. Being abandoned by our friends is the cause of great sorrow for us.

A. Being abandoned by our friends is the cause of great sorrow for us.
B. Our being abandoned by our friends is the cause of great sorrow.
C. Being abandoned by our friends, we feel great sorrow.
D. Abandoned by our friends, sorrow is the result.
E. We feel great sorrow when our friends abandon us.

Correct Answer: E

Explanation:
The constructions using being are awkward and wordy. Answer D is an example of a dangling
modifier. Only Answer E is clear, and in the active voice throughout.

5. Among the many reasons for his defeat in the election was his arrogant assumption that his
constituents were incapable of understanding economic conditions, and his unwarranted attack
on his chief opponent.

A. was his arrogant assumption that his constituents were incapable of understanding economic
conditions
B. were his arrogant assumption that his constituents were incapable of understanding economic
conditions
C. were his arrogant assumptions that his constituents were incapable of understanding
economical conditions
D. were his arrogant assumption that his constituents would be incapable of understanding
economics
E. was the arrogant assumption that his constituents was incapable of understanding economic
conditions

Correct Answer: B

Explanation:

The two reasons were his “arrogant assumption” and his “unwarranted attack”. Since these two
items constitute a compound subject, the verb should be plural (were, not was). In C the word
economical is incorrect.

6. More and more holidaymakers are choosing to fly to remote islands in search of the perfect
beach; seeking sand, sun and palm trees, rather than centers of entertainment.

A. ; seeking sand, sun and palm trees, rather than centers of entertainment.
B. ; seeking sad, sun, palm trees and not entertainment.
C. , with sand, sun, palm trees and no entertainment.
D. , they seek sand, sun and palm trees, rather than entertainment centers.
E. ; they seek sand, sun and palm trees, rather than centers of entertainment.

Correct Answer: E

Explanation:

The correct answer joins two sentences with a semi-colon. A and B use a semi-colon incorrectly.
D needs a semi-colon. C is not parallel.

7. The government requires that these forms should be submitted before the end of the financial
year.
A. that these forms should be submitted
B. that these forms be submitted
C. for these forms to be submitted
D. these forms submission
E. these forms should be submitted

Correct Answer: B

Explanation:

Subjunctive constructions do not need should.

8. After arduous months of fighting, the sight of the white flag being raised generated as much
relief on the victor's side than it did on the vanquished.

A. as much relief on the victor's side than it did on the vanquished.


B. as much relief among the victors as among the vanquished.
C. as much relief on the victor's side as it did on the vanquished's.
D. relief both on the victor's side as well as on the vanquished's.
E. relief both for the victor and the vanquished side.

Correct Answer: B

Explanation:

B has parallel construction and yet is concise.

9. The best way to encourage innovative thinking is not to promise financial rewards for ideas,
but to ensure that the person making the suggestion receives recognition for his contribution.

A. but to ensure that the person making the suggestion receives recognition for his contribution.
B. but to ensure that the person who makes the suggestion will be receiving recognition for his
contribution.
C. but rather by ensuring that the person making the suggestion receives recognition for his
contribution.
D. but rather ensure that suggestion-maker receives recognition for his contribution.
E. but instead make sure that the suggestion-maker will receive recognition.

Correct Answer: A

Explanation:

No error.

10. It ought to be her with whom you share your secrets, not me.
A. her with whom you share your secrets, not me
B. her with whom you share your secrets, not I.
C. she with whom you share your secrets, not me.
D. she with whom you share your secrets, not I.
E. her with who you share your secrets, not me.

Correct Answer: A

Explanation:

After the different forms of the verb to be, we use subject forms of pronouns, except after the
infinitive where we use the object form.
SAT sentence correction practice test 02
1. A conjunction is used to connect words and sentences together.

A. words and sentences together.


B. words or sentences together.
C. words and sentences.
D. words or sentences.
E. words to sentences.

2. Antony, coming alongside Cleopatra's ship, climbed aboard without seeing or being seen by
her.

A. climbed aboard without seeing or being seen by her.


B. climbed aboard without seeing Cleopatra or being seen by her.
C. climbs aboard without Cleopatra seeing him.
D. boarded without being seen by her.
E. boarded without seeing Cleopatra.

3. Bombast is when high sounding words for effect, not suitability, are used.

A. is when high sounding words for effect, not suitability, are used.
B. is the use of high-sounding words for effect rather than for suitability.
C. is where high-sounding words are used for effect not suitability.
D. is the using of high-sounding words for effect only.
E. is when you use high-sounding words for effect rather than for suitability.

4. I would like to thank whoever it was that wrote that piece of music: it has given me so much
pleasure.

A. I would like to thank whoever it was that wrote that piece of music:
B. I would like to thank whomever it was that has written that piece of music:
C. I would like to thank whomever it might be that wrote that piece of music:
D. Whoever it was that wrote that piece of music, I would like to thank because
E. I would like to thank whoever it was that wrote that piece of music,

5. Your taking a loan to buy a car annoyed Father.

A. Your taking a loan to buy a car annoyed Father.


B. Your taking a loan to buy a car aggravated Father.
C. You taking a loan to buy a car irritated Father.
D. You taking a loan to buy a car annoyed Father.
E. Father was annoyed by you taking a loan to buy a car.

6. In the initial stages of learning a new language we learn more through listening and attempting
to copy speech patterns and not through reading grammar books.

A. and attempting to copy speech patterns and not through reading grammar books.
B. and attempting to copy speech patterns than through reading grammar books.
C. and attempts to copy speech patterns than through reading grammar books.
D. and attempts at copying speech patterns than through reading grammar books.
E. and attempts at copying speech patterns and not grammar books.

7. Between you and I, I doubt that he will come.

A. and I, I doubt that he will come.


B. and I, I doubt that he would come.
C. and me, I doubt that he will come.
D. and me, I doubt that he would come.
E. and me, I doubt whether he will be coming.

8. Work that is not finished is not work at all, it is merely a botch, a failure.

A. all, it is merely a botch, a failure.


B. all, it is a botch merely, and a failure.
C. all; it is merely a botch, a failure.
D. all; the work merely is a botch and a failure.
E. all; the work being merely a botch, a failure.
9. Pollution and degradation of the environment is, according to the commission's report, a
matter of universal concern.

A. is, according to the commission's report, a matter of universal concern.


B. is, according to the report by the commission, a matter of universal concern.
C. is, according to the report of the commission, matters of universal concern.
D. are, according to the commission's report, universally a matter of concern.
E. are, according to the commission's report, a matter of universal concern.

10. Denim jeans were originally worn not so much as a fashion statement as for their being
practical work clothes.

A. Denim jeans were originally worn not so much as a fashion statement as for their being
practical work clothes.
B. Denim jeans were originally worn not so much as a fashion statement but for their being
practical work clothes.
C. Denim jeans were originally worn not so much as a fashion statement but for being
practical work clothes
D. Denim jeans were originally worn not as a fashion statement as for them being practical
work clothes.
E. Denim jeans were originally worn not as a fashion statement but as for them being
practical work clothes.

1. A conjunction is used to connect words and sentences together.

A. words and sentences together.


B. words or sentences together.
C. words and sentences.
D. words or sentences.
E. words to sentences.

Correct Answer: D

Explanation:

Together is redundant. The conjunction does not couple words to sentences; it joins words, or it
joins sentences. Hence the meaning is clearest in D.

2. Antony, coming alongside Cleopatra's ship, climbed aboard without seeing or being seen by
her.
A. climbed aboard without seeing or being seen by her.
B. climbed aboard without seeing Cleopatra or being seen by her.
C. climbs aboard without Cleopatra seeing him.
D. boarded without being seen by her.
E. boarded without seeing Cleopatra.

Correct Answer: B

Explanation:

The meaning has to be clear that he did not see her, nor did she see him.

3. Bombast is when high sounding words for effect, not suitability, are used.

A. is when high sounding words for effect, not suitability, are used.
B. is the use of high-sounding words for effect rather than for suitability.
C. is where high-sounding words are used for effect not suitability.
D. is the using of high-sounding words for effect only.
E. is when you use high-sounding words for effect rather than for suitability.

Correct Answer: B

Explanation:

The construction “Bombast is where/when” is incorrect. “Bombast is the using of” is also faulty.

4. I would like to thank whoever it was that wrote that piece of music: it has given me so much
pleasure.

A. I would like to thank whoever it was that wrote that piece of music:
B. I would like to thank whomever it was that has written that piece of music:
C. I would like to thank whomever it might be that wrote that piece of music:
D. Whoever it was that wrote that piece of music, I would like to thank because
E. I would like to thank whoever it was that wrote that piece of music,

Correct Answer: A

Explanation:

No error.

5. Your taking a loan to buy a car annoyed Father.

A. Your taking a loan to buy a car annoyed Father.


B. Your taking a loan to buy a car aggravated Father.
C. You taking a loan to buy a car irritated Father.
D. You taking a loan to buy a car annoyed Father.
E. Father was annoyed by you taking a loan to buy a car.

Correct Answer: A

Explanation:

Do not use aggravated or irritated when you mean annoyed. Also we need a possessive form of
a pronoun in front of a gerund, so you taking is wrong.

6. In the initial stages of learning a new language we learn more through listening and attempting
to copy speech patterns and not through reading grammar books.

A. and attempting to copy speech patterns and not through reading grammar books.
B. and attempting to copy speech patterns than through reading grammar books.
C. and attempts to copy speech patterns than through reading grammar books.
D. and attempts at copying speech patterns than through reading grammar books.
E. and attempts at copying speech patterns and not grammar books.

Correct Answer: B

Explanation:

The word more requires a than. Attempting is correctly parallel to listening.

7. Between you and I, I doubt that he will come.

A. and I, I doubt that he will come.


B. and I, I doubt that he would come.
C. and me, I doubt that he will come.
D. and me, I doubt that he would come.
E. and me, I doubt whether he will be coming.

Correct Answer: C

Explanation:

After the preposition between the pronouns should be in the object form and so we need me and
not I. After doubt the simple future tense is correct.

8. Work that is not finished is not work at all, it is merely a botch, a failure.

A. all, it is merely a botch, a failure.


B. all, it is a botch merely, and a failure.
C. all; it is merely a botch, a failure.
D. all; the work merely is a botch and a failure.
E. all; the work being merely a botch, a failure.

Correct Answer: C

Explanation:

A semicolon is need to link two sentences. Of the choices with the semicolon, C is short and
clear (and correct), D has merely in the wrong place, and E does not have a complete sentence
after the semicolon.

9. Pollution and degradation of the environment is, according to the commission's report, a
matter of universal concern.

A. is, according to the commission's report, a matter of universal concern.


B. is, according to the report by the commission, a matter of universal concern.
C. is, according to the report of the commission, matters of universal concern.
D. are, according to the commission's report, universally a matter of concern.
E. are, according to the commission's report, a matter of universal concern.

Correct Answer: E

Explanation:

“Pollution and degradation” form a compound subject which needs a plural verb: are is correct.

10. Denim jeans were originally worn not so much as a fashion statement as for their being
practical work clothes.

A. Denim jeans were originally worn not so much as a fashion statement as for their being
practical work clothes.
B. Denim jeans were originally worn not so much as a fashion statement but for their being
practical work clothes.
C. Denim jeans were originally worn not so much as a fashion statement but for being practical
work clothes
D. Denim jeans were originally worn not as a fashion statement as for them being practical work
clothes.
E. Denim jeans were originally worn not as a fashion statement but as for them being practical
work clothes.

Correct Answer: A

Explanation:

The construction not so much as... as is correct.


1. It is highly desirable that you furnish evidence of your expenses before you submit your final
accounts.

A. It is highly desirable that you furnish evidence of your expenses

B. It is highly desirable that you should furnish evidence of your expenses

C. It is highly to be desired that you furnish evidences of your expenses

D. You must furnish evidence of your expenses

E. You should have to supply evidence of your expenses

2. The population of tigers in the National Park is increasing steadily, and this is a source of
encouragement to those who have worked so hard to fund the conservation effort.

A. steadily, and this

B. steadily: which

C. steadily; this trend

D. steadily, this increase

E. steady, and this

3. In the fine print at the end of the document lies the clauses that make us liable for any
expenses that result from civil unrest.

A. lies the clauses that make us liable for any expenses that

B. lies the clauses that make us liable for any expenses which

C. lies the clause that make us liable for any expenses that
D. lie the clauses that makes us liable for any expenses which

E. lie the clauses that make us liable for any expenses that

4. The administration discussed whether the number of students studying European languages
was likely to decline when the senior lecturer retired.

A. whether the number of students studying European languages was likely

B. whether the number of students studying European languages were likely

C. if the students studying European languages were likely

D. if the number of European language students were likely

E. whether the number of students studying European languages was liable

5. If the gardener would sow the seeds in the greenhouse rather than the garden, he might get a
better display of flowers.

A. If the gardener would sow the seeds in the greenhouse rather than the garden

B. If the gardener sowed the seeds in the greenhouse rather than the garden

C. If the gardener would sow the seeds in the greenhouse rather than in the garden

D. If the gardener were to sow the seeds in the greenhouse rather than in the garden

E. If the gardener would sow the seeds in the greenhouse instead of the garden

6. On Discovery channel last night they showed an informative program about new innovations
in medical imaging, which you would have found interesting.

A. they showed an informative program about new innovations in medical imaging, which
you would have found interesting.
B. they showed an informative program about innovations in medical imaging, which you
would have found interesting.

C. they showed an informative program about innovations in medical imaging, that you
would have found interesting.

D. there was an informative program about new innovations in medical imaging, that you
would have found interesting.

E. there was an informative program about innovations in medical imaging, a program you
would have found interesting.

7. After working for two hours, the essay started to take shape, and he began to hope that he
might finish before the deadline.

A. After working for two hours, the essay started to take shape, and he began to hope

B. When the essay started to take shape after he had worked on it for two hours, he began to
hope

C. When the essay started to take shape after he had worked on it for two hours, he begun to
hope

D. When the essay started to take shape after working on it for two hours, he began to hope

E. After working for two hours, the student's essay started to take shape, and he began to
hope

8. The young man was surprised to find that his experience as a tutor had been used as the basis
for the protagonist in a short story written by a former girlfriend.

A. that his experience as a tutor had been used as the basis for the protagonist in

B. his experience as a tutor having been used as the protagonist in

C. his experience as a tutor had been used as the basis for the protagonist's in
D. his experience as a tutor being used as the basis for the protagonist of

E. that his experience as a tutor had been used as the basis for events in the life of the
protagonist in

9. In the engineering sector at the moment there are no jobs for those without experience, which
makes it difficult for we recent graduates to get started on our careers.

A. at the moment there are no jobs for those without experience, which makes it difficult for
we

B. at the moment there are no jobs for those without experience, which makes it difficult for
us

C. there are no jobs at the moment for those without experience, a fact that makes it difficult
for us

D. there are no jobs at the moment for those without experience, which makes it difficult for
us

E. there are no jobs at the moment for those without experience, a fact which makes it
difficult for we

10. The mole is a nocturnal insectivorous mammal regarded as pests by gardeners because of
their burrowing activity spoiling lawns and gardens.

A. regarded as pests by gardeners because of their burrowing activity spoiling

B. regarded to be pests by gardeners because of their burrowing activity's spoiling

C. regarded as a pest by gardeners because of burrowing activity spoiling

D. considered as a pest by gardeners because of its burrowing activity spoiling

E. regarded as a pest by gardeners because its burrowing activity spoils lawns and gardens.
1. It is highly desirable that you furnish evidence of your expenses before you submit your final
accounts.

A. It is highly desirable that you furnish evidence of your expenses


B. It is highly desirable that you should furnish evidence of your expenses
C. It is highly to be desired that you furnish evidences of your expenses
D. You must furnish evidence of your expenses
E. You should have to supply evidence of your expenses

Correct Answer: A

Explanation:

No error.

2. The population of tigers in the National Park is increasing steadily, and this is a source of
encouragement to those who have worked so hard to fund the conservation effort.

A. steadily, and this


B. steadily: which
C. steadily; this trend
D. steadily, this increase
E. steady, and this

Correct Answer: C

Explanation:

Both which and this should refer to a specific noun. Since the fist part of the sentence does not
provide a suitable noun, we must introduce a word such as increase or trend. D is incorrect as it
uses a comma to link two sentences.

3. In the fine print at the end of the document lies the clauses that make us liable for any
expenses that result from civil unrest.

A. lies the clauses that make us liable for any expenses that
B. lies the clauses that make us liable for any expenses which
C. lies the clause that make us liable for any expenses that
D. lie the clauses that makes us liable for any expenses which
E. lie the clauses that make us liable for any expenses that

Correct Answer: E

Explanation:
The subject the clauses requires a plural verb; change lies to lie. Similarly, make is required to
agree with clauses.

4. The administration discussed whether the number of students studying European languages
was likely to decline when the senior lecturer retired.

A. whether the number of students studying European languages was likely


B. whether the number of students studying European languages were likely
C. if the students studying European languages were likely
D. if the number of European language students were likely
E. whether the number of students studying European languages was liable

Correct Answer: A

Explanation:

"Whether" is correct because the question concerns a choice not a condition. With the expression
"the number of" a singular verb is needed and hence "was" is correct. "Liable" is used in
expressions such as "liable to prosecution" and not for expressions of possibility.

5. If the gardener would sow the seeds in the greenhouse rather than the garden, he might get a
better display of flowers.

A. If the gardener would sow the seeds in the greenhouse rather than the garden
B. If the gardener sowed the seeds in the greenhouse rather than the garden
C. If the gardener would sow the seeds in the greenhouse rather than in the garden
D. If the gardener were to sow the seeds in the greenhouse rather than in the garden
E. If the gardener would sow the seeds in the greenhouse instead of the garden

Correct Answer: D

Explanation:

The conditional clause (the “if” clause) does not require would. Also we should write “in the
garden” to be parallel to “in the greenhouse”.

6. On Discovery channel last night they showed an informative program about new innovations
in medical imaging, which you would have found interesting.

A. they showed an informative program about new innovations in medical imaging, which you
would have found interesting.
B. they showed an informative program about innovations in medical imaging, which you would
have found interesting.
C. they showed an informative program about innovations in medical imaging, that you would
have found interesting.
D. there was an informative program about new innovations in medical imaging, that you would
have found interesting.
E. there was an informative program about innovations in medical imaging, a program you
would have found interesting.

Correct Answer: E

Explanation:

They is incorrect when talking of the television channel. New is redundant. And which must have
a clear antecedent. Only answer E corrects all these problems.

7. After working for two hours, the essay started to take shape, and he began to hope that he
might finish before the deadline.

A. After working for two hours, the essay started to take shape, and he began to hope
B. When the essay started to take shape after he had worked on it for two hours, he began to
hope
C. When the essay started to take shape after he had worked on it for two hours, he begun to
hope
D. When the essay started to take shape after working on it for two hours, he began to hope
E. After working for two hours, the student's essay started to take shape, and he began to hope

Correct Answer: B

Explanation:

Answers A and E start with a dangling modifier. Answer C uses begun instead of began and D is
unclear because we do not know who is working.

8. The young man was surprised to find that his experience as a tutor had been used as the basis
for the protagonist in a short story written by a former girlfriend.

A. that his experience as a tutor had been used as the basis for the protagonist in
B. his experience as a tutor having been used as the protagonist in
C. his experience as a tutor had been used as the basis for the protagonist's in
D. his experience as a tutor being used as the basis for the protagonist of
E. that his experience as a tutor had been used as the basis for events in the life of the protagonist
in

Correct Answer: E

Explanation:

The tutor was the basis for the protagonist, or the tutor's experience was the basis for the
experience of the protagonist. Only E makes this relation clear.
9. In the engineering sector at the moment there are no jobs for those without experience, which
makes it difficult for we recent graduates to get started on our careers.

A. at the moment there are no jobs for those without experience, which makes it difficult for we
B. at the moment there are no jobs for those without experience, which makes it difficult for us
C. there are no jobs at the moment for those without experience, a fact that makes it difficult for
us
D. there are no jobs at the moment for those without experience, which makes it difficult for us
E. there are no jobs at the moment for those without experience, a fact which makes it difficult
for we

Correct Answer: C

Explanation:

The object us is required after the preposition for. Which requires a clear antecedent. Only C
corrects both problems.

10. The mole is a nocturnal insectivorous mammal regarded as pests by gardeners because of
their burrowing activity spoiling lawns and gardens.

A. regarded as pests by gardeners because of their burrowing activity spoiling


B. regarded to be pests by gardeners because of their burrowing activity's spoiling
C. regarded as a pest by gardeners because of burrowing activity spoiling
D. considered as a pest by gardeners because of its burrowing activity spoiling
E. regarded as a pest by gardeners because its burrowing activity spoils lawns and gardens.

Correct Answer: E

Explanation:

The mole is singular and hence we need its not their. After considered we do not need as.
SAT editing in context practice test 01
Questions 1-6 refer to the following passage, which is a draft of an essay:

(1)Cicero said, “A room without a book is a body without a soul.” (2) Certainly when I enter
someone's home for the first time, I am likely to gravitate to the bookshelf, in part to glean
further insight into the personality of its owner. (3) But now that the family encyclopedia is
likely to have been replaced by a CD-ROM it is possible that the book might be reduced to an
item of decoration rather than information and entertainment.

(4)In a sense, books have always been more than just repositories of information. (5) The look
and feel of a book is as much a part of its appeal as its contents. (6) There is something
immensely satisfying about opening a new book: the smell of the paper, the feel of the cover, the
design on the dust jacket and the weight of the volume all contribute to the impression it makes.
(7) The most aesthetically pleasing volumes, the leather bound volumes, and the volumes with
beautiful bindings are actually often bought by interior decorators to add to the look of a study,
office or of a living room.

(8)Books have a symbolic power. We shudder when we hear of ‘book burning', associated down
the ages with tyranny and oppression. (9) Books as cultural icons remind us of freedom of
speech and enhanced opportunities, they remind us of the intellectual aspirations of the human
race.

(10)But in the future will the book still be read? (11)I believe it will. (12)More books are being
written and published than ever before; the book has withstood the advent of the cinema,
television, and personal computer and are likely to be there in centuries to come.

1. Which is the best version of the underlined portion of sentence 3 (reproduced below)?
But now that the family encyclopedia is likely to have been replaced by a CD-ROM it is possible
that the book might be reduced to an item of decoration rather than information and
entertainment.

A. might be reduced to an item of decoration rather than information


B. might be reduced to an item of decoration rather than a source of information
C. will be reduced to an item of decoration rather than information
D. will be reduced to an item of decoration rather than a source of information
E. could be reduced to an item of decoration rather than information

2. Which version of sentence 8 would form the best transition between paragraphs two and
three?
A. Besides their aesthetic appeal, books have symbolic power.
B. Books have also always had symbolic power.
C. Besides their use to decorators, books have a symbolic power.
D. Books have other functions besides decoration.
E. Other people have used books as symbols.

3. What revision is most needed in sentence 9?

A. Change “enhanced” to “equal”.


B. Change the comma to a semicolon.
C. Change “us” to “people”
D. Rewrite to avoid repetition of “they remind us of”.
E. Insert “Moreover” at the beginning of the sentence.

4. Sentence 7 would probably benefit from all the following changes EXCEPT

A. Change from passive to active voice


B. Correct faulty parallelism
C. Rewrite to avoid repetition of “volumes”.
D. Delete the word “actually”.
E. Remove the word “aesthetically”.

5. Paragraph three

A. is somewhat underdeveloped
B. is redundant
C. is too repetitive
D. contains error in verb tense
E. has errors in pronoun usage

6. Which is the best version of the underlined part of the last sentence (reproduced below)?
More books are being written and published than ever before; the book has withstood the advent
of the cinema, television, and personal computer and are likely to be there in centuries to come.

A. the book has withstood the advent of the cinema, television, personal computer and are
likely
B. the book has withstood the advent of the cinema, television, and personal computer and
are liable
C. books have withstood the advent of the cinema, television, and personal computer and are
likely
D. the book withstood the advent of the cinema, television, personal computer and is likely
E. the book withstood the advent of the cinema, television, and personal computer and will
likely

Questions 7-12 refer to the following passage, which is a draft of an essay:

(1)The study of ecology has taught us that diversity is important to stability in the natural world.
(2)Modern agricultural practices and other human interventions in the environment reduces the
number of interacting species making the ecosystem vulnerable and unstable.

(3)Similarly, human society in its progress toward the clichéd “global village” is liable to make
the world less stable not more stable. (4)We are eliminating languages, traditional diets, and
eroding cultural practices at an alarming rate. (5)The whole world wants to speak the same
language, eat the same food and wear the same clothes. (6)But will this homogeneity be good for
the human species? (7) Biologists would tend to say no.

(8)Certainly a world where we can all understand each other and share common interests is
appealing on one level. (9)But how infinitely poorer we will all be if we travel 5000 miles from
home only to find no new people, no new places, no new ways of living, just a copy of what you
experience back home.

7. One weakness of the writing style of this essay is

A. Reliance on general statements


B. Incorrect paragraphing
C. Inappropriate vocabulary
D. Poor punctuation
E. Poor choice of verb tenses

8. The author makes his point about human society mainly by use of

A. personal experience
B. scientific facts
C. parallel construction
D. an analogy
E. hyperbole

9. Sentence 2 could be best improved by

A. insert a comma after “environment”


B. remove the word “other”
C. remove the word “interacting”
D. change “interventions” to “intervention”
E. change “reduces” to “reduce”

10. Which is the best version of the underlined part of sentence 3 (reproduced below)?
Similarly, human society in its progress toward the clichéd “global village” is liable to make the
world less stable not more stable.

A. is likely to make the world less stable not


B. is likely to make the world not less stable but
C. is likely to make the world less and not
D. are liable to make the world less stable not
E. are likely to make the world less stable not

11. The best version of sentence four is

A. We are eliminating languages, traditional diets, and eroding cultural practices at an


alarming rate.
B. We are eliminating languages, changing traditional diets, and cultural practices at an
alarming rate.
C. We are eliminating languages, changing traditional diets, and eroding cultural practices at
an alarming rate.
D. Languages, traditional diets, and cultural practices are being eroded at an alarming rate.
E. Languages, traditional diets, and cultural practices are being eliminated alarmingly fast.

12. The change most needed in sentence 9 is to replace

A. will with would


B. you with we
C. just with only
D. infinitely with much
E. experience to experienced

SAT editing in context explanation


Test 01
1. Which is the best version of the underlined portion of sentence 3 (reproduced below)?
But now that the family encyclopedia is likely to have been replaced by a CD-ROM it is possible
that the book might be reduced to an item of decoration rather than information and
entertainment.

A. might be reduced to an item of decoration rather than information


B. might be reduced to an item of decoration rather than a source of information
C. will be reduced to an item of decoration rather than information
D. will be reduced to an item of decoration rather than a source of information
E. could be reduced to an item of decoration rather than information

Correct Answer: D

Explanation:

A future tense is needed after possible. Also parallel construction is needed around rather than.

2. Which version of sentence 8 would form the best transition between paragraphs two and
three?

A. Besides their aesthetic appeal, books have symbolic power.


B. Books have also always had symbolic power.
C. Besides their use to decorators, books have a symbolic power.
D. Books have other functions besides decoration.
E. Other people have used books as symbols.

Correct Answer: A

Explanation:

This sentence indicates the transition from writing about the aesthetic aspects of the book to its
use as a symbol.

3. What revision is most needed in sentence 9?


A. Change “enhanced” to “equal”.
B. Change the comma to a semicolon.
C. Change “us” to “people”
D. Rewrite to avoid repetition of “they remind us of”.
E. Insert “Moreover” at the beginning of the sentence.

Correct Answer: B

Explanation:

The comma splice error has to be corrected.

4. Sentence 7 would probably benefit from all the following changes EXCEPT

A. Change from passive to active voice


B. Correct faulty parallelism
C. Rewrite to avoid repetition of “volumes”.
D. Delete the word “actually”.
E. Remove the word “aesthetically”.

Correct Answer: E

Explanation:

The word aesthetically helps reinforce the point the writer is making in this paragraph and so
should be retained.

5. Paragraph three

A. is somewhat underdeveloped
B. is redundant
C. is too repetitive
D. contains error in verb tense
E. has errors in pronoun usage

Correct Answer: A

Explanation:

The author could have elucidated the idea of the book as symbol, could have clarified his point
about book-burning, and could have given a specific illustration.

6. Which is the best version of the underlined part of the last sentence (reproduced below)?
More books are being written and published than ever before; the book has withstood the advent
of the cinema, television, and personal computer and are likely to be there in centuries to come.
A. the book has withstood the advent of the cinema, television, personal computer and are likely
B. the book has withstood the advent of the cinema, television, and personal computer and are
liable
C. books have withstood the advent of the cinema, television, and personal computer and are
likely
D. the book withstood the advent of the cinema, television, personal computer and is likely
E. the book withstood the advent of the cinema, television, and personal computer and will likely

Correct Answer: C

Explanation:

In answer A are is incorrect; in answer B liable is incorrect; in E will likely is wrong. C is better
than D because it uses the present perfect tense and it uses the plural books as does the first part
of the sentence.

7. One weakness of the writing style of this essay is

A. Reliance on general statements


B. Incorrect paragraphing
C. Inappropriate vocabulary
D. Poor punctuation
E. Poor choice of verb tenses

Correct Answer: A

Explanation:

An essay without specific detail and supporting facts does not make its point effectively.

8. The author makes his point about human society mainly by use of

A. personal experience
B. scientific facts
C. parallel construction
D. an analogy
E. hyperbole

Correct Answer: D

Explanation:

The author uses the need for diversity in the ecosystem as an analogy for the need for diversity in
society.

9. Sentence 2 could be best improved by


A. insert a comma after “environment”
B. remove the word “other”
C. remove the word “interacting”
D. change “interventions” to “intervention”
E. change “reduces” to “reduce”

Correct Answer: E

Explanation:

A plural subject requires a plural verb.

10. Which is the best version of the underlined part of sentence 3 (reproduced below)?
Similarly, human society in its progress toward the clichéd “global village” is liable to make the
world less stable not more stable.

A. is likely to make the world less stable not


B. is likely to make the world not less stable but
C. is likely to make the world less and not
D. are liable to make the world less stable not
E. are likely to make the world less stable not

Correct Answer: A

Explanation:

Use likely not liable.

11. The best version of sentence four is

A. We are eliminating languages, traditional diets, and eroding cultural practices at an alarming
rate.
B. We are eliminating languages, changing traditional diets, and cultural practices at an alarming
rate.
C. We are eliminating languages, changing traditional diets, and eroding cultural practices at an
alarming rate.
D. Languages, traditional diets, and cultural practices are being eroded at an alarming rate.
E. Languages, traditional diets, and cultural practices are being eliminated alarmingly fast.

Correct Answer: C

Explanation:

C has the best parallel construction.

12. The change most needed in sentence 9 is to replace


A. will with would
B. you with we
C. just with only
D. infinitely with much
E. experience to experienced

Correct Answer: B

Explanation:

You disrupts the pronoun continuity.


SAT editing in context practice test 02
Questions 1-6 refer to the following passage, which is a draft of an essay:

(1)I recently revisited the city which I was born in, which is a place well known for a castle built
on a rock overlooking the surrounding plains, and even better known for a legendary figure who
robbed the rich to give to the poor. (2)As I toured the castle and its museum, visited the town
center, and roamed around old haunts, I reflected on how the buildings that people of different
eras build reflect their central preoccupations.

(3)The castle was originally built in the eleventh century, and remained important for several
centuries. (4)Throughout the medieval period castles and fortified houses were built. (5)The
powerful landowners surveyed and dominated the surrounding lands, the source of their wealth
and prestige.

(6)Once the industrial era began, castles were sidelined. (7)The merchants and factory owners
built town halls, churches, factories and imposing office buildings. (7)The town center reflects
this era. (8)An imposing town hall, complete with massive pillars and monumental lions,
overlooks the town square. (9)The square is surrounded by equally massive blocks of shops,
banks and offices, built to reflect the power of trade in the heyday of the British Empire.

(10)Most people who enter the city today never visit the castle or the old market square, they
head for the two shopping malls situated at either end of the city center. (11)Here the visitor can
shop to their hearts content in an environment of glass and polished chrome. (12)Modern man is
no longer a warrior defending his land, or a builder of churches, or a governor of people: he is
primarily a consumer. (13)The buildings our generation leaves to posterity will reflect our
predominant interest – shopping.  

1. The author's approach to the topic can best be described as

A. rhetorical
B. a reminiscence
C. a specific example to illustrate a general point
D. personal narrative
E. several examples used to contradict a viewpoint

2. The sentence (or sentences) which most clearly expresses the author's primary purpose is

A. (1)
B. (2)
C. (12)
D. (13)
E. (1) and (13)

3. Which is the best version of the underlined portion of sentence (1), (reproduced below)? 
I recently revisited the city which I was born in, which is a place well known for a castle built on
a rock overlooking the surrounding plains, and even better known for a legendary figure who
robbed the rich to give to the poor.

A. I recently revisited the city which I was born in, which is a place
B. I recently revisited the city in which I was born in, which is
C. I recently revisited the city in which I was born, a city
D. Recently I revisited the place which I was born in, which is a city
E. Recently I revisited the city which I was born in, which is

4. The best way to rewrite the underlined parts of sentences 4 and 5 (reproduced below) in order
to combine the sentences is:
Throughout the medieval period castles and fortified houses were built. The powerful
landowners surveyed and dominated the surrounding lands, the source of their wealth and
prestige.

A. Throughout the medieval period castles and fortified houses were built from which
powerful landowners surveyed and dominated
B. Castles and fortified houses were built throughout the medieval period, from which
powerful landowners surveyed and dominated
C. Castles and fortified houses were built throughout the medieval period, when powerful
landowners surveyed and dominated
D. Throughout the medieval period, powerful landowners built castles and fortified houses
from which to survey and dominate
E. Throughout the medieval period, powerful landowners built castles and fortified houses
from which they would have surveyed and dominated

5. The change most needed in sentence 10 is

A. change or to nor
B. change “either end” to “both ends”
C. change the comma to a semicolon
D. avoid the repetition of the word “city”
E. change people to persons

6. Sentence 11 has faulty

A. pronoun usage
B. parallelism
C. verb tenses
D. diction
E. verb agreement

Questions 7-12 refer to the following passage, which is a draft of an essay:

(1)Nowadays we tend to use the expression “plain vanilla” in a derogatory sense for something
simple and unadorned. (2)But have you ever thought about the ways in which the vanilla bean
has improved the taste of what we eat?

(3)One of the most used, and presumably, most appreciated, flavors known to man, worldwide
we use more than one thousand tons of vanilla beans every year, mainly to flavor ice cream,
cakes and confectionery. (4)Since the 1930s chemical companies have also been making huge
amounts of synthetic vanilla essence mainly from the byproducts of the paper pulp industry.
(5)In fact, most of the “vanilla flavor” products on the supermarket shelves have never seen a
vanilla pod. (6)However, good cooks keep real vanilla pods in their sugar to enhance the taste of
their cakes and cookies and would not buy synthetic vanilla.

(7)Natural vanilla is the product of a creeper that grows best in hot wet climates. (8)It requires
hand pollinating and the growing, harvesting and curing need skill and are labor intensive.
(9)With the advent of biotechnology it remains to be seen how long the traditional growing and
production methods would survive.

(10)But whatever the source, vanilla has become essential to the human palate. (11)Think about
it: do you even know what “plain” ice cream would taste like without vanilla?

7. The writer uses all of the following techniques except

A. rhetorical questions
B. statement of facts
C. speculation
D. address to the reader
E. personal experience
8. Which version of the underlined part best corrects and improves sentence 3 (reproduced
below)?
One of the most used, and presumably, most appreciated, flavors known to man, worldwide we
use more than one thousand tons of vanilla beans every year, mainly to flavor ice cream, cakes
and confectionery.

A. Vanilla is certainly one of the most used, and presumably most appreciated, flavors
known to man. Worldwide
B. Vanilla is certainly one of the most used, and presumably most appreciated, flavors
known to man, worldwide
C. Vanilla is certainly one of the most used, and presumably most appreciated, flavors
known to man worldwide:
D. One of the most used, and presumably most appreciated, flavors known to man
worldwide, vanilla
E. Worldwide vanilla is certainly one of the most used, and presumably most appreciated,
flavors known to man.

9. The best replacement for the word “it” in sentence 8 is

A. vanilla
B. the pod
C. vanilla flowers
D. the plant
E. pods

10. The change most needed in sentence 9 is

A. change “would” to “will”


B. change “methods” to “method”
C. rewrite to avoid “to be”
D. insert “for” in front of “how”
E. substitute “arrival” for “advent”

11. One reason the writer uses the word “plain” in the last sentence is probably

A. to contrast with “vanilla”


B. to link to the use of the same word in the first sentence
C. to emphasis an opposite
D. to clarify what is meant by flavor
E. to add more force to the argument

12. Which sentence best states the author's contention about vanilla?

A. 1
B. 2
C. 9
D. 10
E. 11

SAT editing in context explanation


Test 02
1. The author's approach to the topic can best be described as

A. rhetorical
B. a reminiscence
C. a specific example to illustrate a general point
D. personal narrative
E. several examples used to contradict a viewpoint

Correct Answer: C

Explanation:

The author uses the examples of his/her home city to make the point that buildings reflect the
concerns of an era

2. The sentence (or sentences) which most clearly expresses the author's primary purpose is

A. (1)
B. (2)
C. (12)
D. (13)
E. (1) and (13)

Correct Answer: B
Explanation:

The author intends to show how the different buildings reflect the concerns of different ages.

3. Which is the best version of the underlined portion of sentence (1), (reproduced below)? 
I recently revisited the city which I was born in, which is a place well known for a castle built on
a rock overlooking the surrounding plains, and even better known for a legendary figure who
robbed the rich to give to the poor.

A. I recently revisited the city which I was born in, which is a place
B. I recently revisited the city in which I was born in, which is
C. I recently revisited the city in which I was born, a city
D. Recently I revisited the place which I was born in, which is a city
E. Recently I revisited the city which I was born in, which is

Correct Answer: C

Explanation:

Answer (C) is the most concise and avoids the awkward repetition of which.

4. The best way to rewrite the underlined parts of sentences 4 and 5 (reproduced below) in order
to combine the sentences is:
Throughout the medieval period castles and fortified houses were built. The powerful
landowners surveyed and dominated the surrounding lands, the source of their wealth and
prestige.

A. Throughout the medieval period castles and fortified houses were built from which powerful
landowners surveyed and dominated
B. Castles and fortified houses were built throughout the medieval period, from which powerful
landowners surveyed and dominated
C. Castles and fortified houses were built throughout the medieval period, when powerful
landowners surveyed and dominated
D. Throughout the medieval period, powerful landowners built castles and fortified houses from
which to survey and dominate
E. Throughout the medieval period, powerful landowners built castles and fortified houses from
which they would have surveyed and dominated

Correct Answer: D

Explanation:

Answers A, B and C all use the passive voice. Of choices D and E, which use the active voice, D
is the more concise.

5. The change most needed in sentence 10 is


A. change or to nor
B. change “either end” to “both ends”
C. change the comma to a semicolon
D. avoid the repetition of the word “city”
E. change people to persons

Correct Answer: C

Explanation:

The major grammar error is the use of the comma to link two full sentences. Other changes are
matters of style.

6. Sentence 11 has faulty

A. pronoun usage
B. parallelism
C. verb tenses
D. diction
E. verb agreement

Correct Answer: A

Explanation:

A visitor is singular and so the sentence should state "to his or her heart's content".

7. The writer uses all of the following techniques except

A. rhetorical questions
B. statement of facts
C. speculation
D. address to the reader
E. personal experience

Correct Answer: E

Explanation:

The author does not give her own experience but makes general statements.

8. Which version of the underlined part best corrects and improves sentence 3 (reproduced
below)?
One of the most used, and presumably, most appreciated, flavors known to man, worldwide we
use more than one thousand tons of vanilla beans every year, mainly to flavor ice cream, cakes
and confectionery.
A. Vanilla is certainly one of the most used, and presumably most appreciated, flavors known to
man. Worldwide
B. Vanilla is certainly one of the most used, and presumably most appreciated, flavors known to
man, worldwide
C. Vanilla is certainly one of the most used, and presumably most appreciated, flavors known to
man worldwide:
D. One of the most used, and presumably most appreciated, flavors known to man worldwide,
vanilla
E. Worldwide vanilla is certainly one of the most used, and presumably most appreciated, flavors
known to man.

Correct Answer: A

Explanation:

The original version has a dangling modifier, which needs to be corrected.  “Worldwide” refers
to the use of vanilla. Sentence A deals with both problems.

9. The best replacement for the word “it” in sentence 8 is

A. vanilla
B. the pod
C. vanilla flowers
D. the plant
E. pods

Correct Answer: D

Explanation:

A singular subject is required. We are referring to the creeper, which is a plant.

10. The change most needed in sentence 9 is

A. change “would” to “will”


B. change “methods” to “method”
C. rewrite to avoid “to be”
D. insert “for” in front of “how”
E. substitute “arrival” for “advent”

Correct Answer: A

Explanation:

A simple future tense is needed.


11. One reason the writer uses the word “plain” in the last sentence is probably

A. to contrast with “vanilla”


B. to link to the use of the same word in the first sentence
C. to emphasis an opposite
D. to clarify what is meant by flavor
E. to add more force to the argument

Correct Answer: B

Explanation:

The author starts with the idea of “plain vanilla” and rounds of by returning to the same idea.

12. Which sentence best states the author's contention about vanilla?

A. 1
B. 2
C. 9
D. 10
E. 11

Correct Answer: D

Explanation:

The author is making a strong claim that vanilla is now essential. (Whether she has adequately
justified her contention is another matter.)
Word Focus Word Associations test 01
Questions 1-10 are Antonyms. Find the word most nearly OPPOSITE in meaning to the given
word.

1. Whet is most nearly opposite in meaning to

A. Deaden
B. Engender
C. Default
D. Enhance
E. Desiccate

2. Obstreperous is most nearly opposite in meaning to

A. Placid
B. Stubborn
C. Raucous
D. Febrile
E. Amicable

3. Peripheral is most nearly opposite in meaning to

A. Astute
B. Central
C. Concrete
D. Axial
E. Radial

4. Ossified is most nearly opposite in meaning to

A. Durable
B. Dainty
C. Flexible
D. Contumacious
E. Pragmatic

5. Disingenuous is most nearly opposite in meaning to

A. Transparent
B. Parched
C. Blank
D. Ingratiating
E. Clever

6. Disheveled is most nearly opposite in meaning to

A. Crumpled
B. Angelic
C. Fermented
D. Demented
E. Ordered

7. Duplicitous is most nearly opposite in meaning to

A. Focused
B. Jaundiced
C. Monomaniacal
D. Straightforward
E. Approachable

8. Untoward is most nearly opposite in meaning to

A. Proactive
B. Abstemious
C. Expected
D. Egregious
E. Tortuous
9. Unequivocal is most nearly opposite in meaning to

A. Multifaceted
B. Ambiguous
C. Unanimous
D. Miniscule
E. Polemical

10. Enervating is most nearly opposite in meaning to

A. Idyllic
B. Grandiose
C. Decorous
D. Frustrating
E. Invigorating

Questions 11-20 are Synonyms. Find the word most SIMILAR in meaning to the given word.

11. Unprepossessing is most similar in meaning to

A. Ugly
B. Arrogant
C. Personable
D. Didactic
E. Sumptuous

12. Turbid is most similar in meaning to

A. Shiny
B. Murky
C. Pellucid
D. Petrified
E. Agitated

13. Potable is most similar in meaning to


A. Bland
B. Unsavory
C. Drinkable
D. Fertile
E. Distilled

14. Flagrant is most similar in meaning to

A. Inauthentic
B. Unperturbed
C. Slanderous
D. Covert
E. Blatant

15. Undermine is most similar in meaning to

A. Impose
B. Apprehend
C. Glorify
D. Weaken
E. Tunnel

16. Tenuous is most similar in meaning to

A. Dark
B. Playful
C. Weak
D. Unfathomable
E. Direct

17. Parochial is most similar in meaning to

A. Cosmopolitan
B. Xenophobic
C. Parsimonious
D. Belligerent
E. Conventional

18. Destitute is most similar in meaning to

A. Impoverished
B. Desolate
C. Affluent
D. Meticulous
E. Derelict

19. Tacit is most similar in meaning to

A. Tenacious
B. Ostensible
C. Garrulous
D. Peaceful
E. Unspoken

20. Palliative is most similar in meaning to

A. Tyrannical
B. Overt
C. Curative
D. Festive
E. Restive

Questions 21-25 are sentence completions. Choose the answer which contains the words that best
fit the blanks and complete the meaning of the given sentence.

21. Federer gave a ____ performance in the final; the spectators were treated to a display of his
____ abilities on the court.

A. stupendous - inconsolable
B. consummate - peerless
C. disappointing - incomparable
D. competent - waning
E. stellar - limited

22. The critics ____ remarks helped us to understand the implications of this ____ film, a film
that was really ground-breaking in its approach.

A. instructive - pedestrian
B. polemical - provocative
C. pertinent - avant-garde
D. unwarranted - conservative
E. helpful - innocuous

23. The Chaplin films that she watched when young left a(n) ____ impression on Suzie, and she
now has a ____ appetite for reading anything remotely related to Chaplin’s life and works.

A. overwhelming - blunted
B. forcible - terrible
C. negative - sharpened
D. indelible - voracious
E. evanescent - reduced

24. The lawyer was ____ for conduct that was in complete violation of legal procedures: he was
found to have ____ a member of the jury.

A. reprimanded - observed
B. disbarred - suborned
C. expelled - questioned
D. commended - mislead
E. disqualified - rejected

25. The management had to admit to clandestine activities when the union leader was able to
supply ____ evidence that an agitator had been planted among the members in order to ____
trouble and discredit the union.

A. egregious - avert
B. undeniable - mollify
C. circumstantial - prevent
D. unsubstantiated - instigate
E. incontrovertible - foment

Word Associations test results


Practice Test 01
Question Your Answer Correct Answer Result

1. A A Correct

2. A A Correct

3. A B Wrong

4. A C Wrong

5. A A Correct

6. A E Wrong

7. A D Wrong

8. A C Wrong

9. A B Wrong

10. A E Wrong

11. A A Correct

12. A B Wrong

13. A C Wrong

14. A E Wrong

15. A D Wrong

16. A C Wrong

17. A B Wrong

18. A A Correct

19. A E Wrong
20. A C Wrong

21. A B Wrong

22. A C Wrong

23. A D Wrong

24. A B Wrong

25. A E Wrong

Your score: 5 out of a possible 25


Word Focus Word Associations test 02
Questions 1-10 are Antonyms. Find the word most nearly OPPOSITE in meaning to the given
word.

1. Apocryphal is most nearly opposite in meaning to

A. Doubtful in nature
B. Accepted as genuine
C. Wide in scope
D. Completely alien
E. Baseless

2. Eschew is most nearly opposite in meaning to

A. Embrace
B. Gulp
C. Reject
D. Transgress
E. Follow

3. Unwarranted is most nearly opposite in meaning to

A. Guaranteed
B. Justified
C. Practiced
D. Jaundiced
E. Tranquil

4. Hoary is most nearly opposite in meaning to

A. Soft
B. Grainy
C. Alert
D. Genuine
E. Novel

5. Allay is most nearly opposite in meaning to

A. Guide
B. Mislead
C. Intensify
D. Reject
E. Reinstate

6. Noxious is most nearly opposite in meaning to

A. Affirmative
B. Diurnal
C. Pungent
D. Deliberate
E. Innocuous

7. Inveterate is most nearly opposite in meaning to

A. Advanced
B. Naïve
C. Habitual
D. Infrequent
E. Professional

8. Vagaries is most nearly opposite in meaning to

A. Certainties
B. Depressions
C. Miracles
D. Probabilities
E. Severances
9. Inclement is most nearly opposite in meaning to

A. Reverent
B. Generous
C. Unmerciful
D. Mortal
E. Favorable

10. Grandiose is most nearly opposite in meaning to

A. Polished
B. Restrained
C. Unstinting
D. Enigmatic
E. Sonorous

Questions 11-20 are Synonyms. Find the word most SIMILAR in meaning to the given word.

11. Sedentary is most similar in meaning to

A. Juvenile
B. Disgraced
C. Tranquil
D. Treacherous
E. Inactive

12. Entrenched is most similar in meaning to

A. Established
B. Disinterred
C. Conflicting
D. Loud
E. Relevant

13. Resolute is most similar in meaning to


A. Praiseworthy
B. Firm
C. Controlled
D. Justified
E. Complete

14. Mollify is most similar in meaning to

A. Stretch
B. Convert
C. Pacify
D. Appraise
E. Unite

15. Virtuoso is most similar in meaning to

A. Talented
B. Inept
C. Upright
D. Silent
E. Verdant

16. Ambivalent is most similar in meaning to

A. Double
B. Fearsome
C. Broad
D. Uncertain
E. Adult

17. Irrevocable is most similar in meaning to

A. Mocking
B. Vicious
C. Puerile
D. Ineffable
E. Permanent

18. Pernicious is most similar in meaning to

A. Avaricious
B. Quarrelsome
C. Dangerous
D. Infinite
E. Fussy

19. Trounce is most similar in meaning to

A. Terrorize
B. Defeat
C. Vibrate
D. Hypnotize
E. Deafen

20. Variegated is most similar in meaning to

A. Dishonest
B. Decorous
C. Monotonous
D. Multicolored
E. Patterned

Questions 21-25 are sentence completions. Choose the answer which contains the words that best
fit the blanks and complete the meaning of the given sentence.

21. The scientist’s enthusiasm was ____ ; she had already published enough papers on the topic
to fill a ____, and yet there were no definitive conclusions to be drawn, and the work seemed
interminable.

A. waning - paragraph
B. flagging - tome
C. unqualified - journal
D. at a peak - book
E. increasing - library

22. There was no need to read the ____ : we knew only one journalist who dared to use such
____ expressions in a publication known for its formal language.

A. work - conservative
B. title - cryptic
C. publication - grotesque
D. byline - colloquial
E. article - strong

23. It is true that ageing is an ____ process, yet there no need to ____ despair: there is much that
we can do to delay the worst effects of old age.

A. upsetting - refuse to
B. interminable - agree to
C. inexorable - wallow in
D. inevitable - gloss over
E. uplifting - succumb to

24. The exorbitantly expensive décor of the mansion was widely criticized: the celebrity’s ____
display of wealth ____ all the norms of good taste.

A. ostentatious - transcended
B. avaricious - upheld
C. grandiose - underpinned
D. restrained - subverted
E. excessive - confirmed

25. The bully had to retreat ____ from the playground when his victim remained ____ in the face
of his taunts; the firmness of the former victim rallied the support of the other children who
turned on the former bully with contempt.
A. rapidly - passive
B. ignominiously - implacable
C. repeatedly - cowed
D. infrequently - intimidated
E. immediately - silent

Word Associations test results


Practice Test 02
Question Your Answer Correct Answer Result

1. A B Wrong

2. A A Correct

3. A B Wrong

4. A E Wrong

5. A C Wrong

6. A E Wrong

7. A D Wrong

8. A A Correct

9. A E Wrong

10. A B Wrong

11. A E Wrong

12. A A Correct

13. A B Wrong

14. A C Wrong

15. A A Correct

16. A D Wrong

17. A E Wrong
18. A C Wrong

19. A B Wrong

20. A D Wrong

21. A B Wrong

22. A D Wrong

23. A C Wrong

24. A A Correct

25. A B Wrong

Your score: 5 out of a possible 25


Word Focus Synonym Sets test 01
Questions 1-15 are Antonyms. Find the word most nearly OPPOSITE in meaning to the given
word.

1. Periphrastic is most nearly opposite in meaning to

A. Concise
B. Circular
C. Comprehensive
D. Ardent
E. Junior

2. Sprightly is most nearly opposite in meaning to

A. Voluble
B. Nimble
C. Sluggish
D. Hurried
E. Genuine

3. Taciturn is most nearly opposite in meaning to

A. Spoken
B. Placatory
C. Leisurely
D. Talkative
E. Confident

4. Prolix is most nearly opposite in meaning to

A. Antediluvian
B. Terse
C. Awkward
D. Belligerent
E. Forward

5. Revere is most nearly opposite in meaning to

A. Accept
B. Convert
C. Accelerate
D. Consider
E. Vilify

6. Plaudit is most nearly opposite in meaning to

A. Denunciation
B. Pleasure
C. Honorary
D. Disgrace
E. Diffidence

7. Desultory is most nearly opposite in meaning to

A. Salacious
B. Cursory
C. Assiduous
D. Asinine
E. Jovial

8. Blundering is most nearly opposite in meaning to

A. Arousing
B. Shocking
C. Disgraceful
D. Nimble
E. Nonchalant
9. Castigate is most nearly opposite in meaning to

A. Incarcerate
B. Extol
C. Remit
D. Flatten
E. Change

10. Salute is most nearly opposite in meaning to

A. Acquire
B. Cogitate
C. Espy
D. Plant
E. Defame

11. Lackadaisical is most nearly opposite in meaning to

A. Conscientious
B. Superficial
C. Abrupt
D. Egotistical
E. Germane

12. Fastidious is most nearly opposite in meaning to

A. Monotonous
B. Negligent
C. Interminable
D. Aloof
E. Miniscule

13. Execrate is most nearly opposite in meaning to


A. Inter
B. Devolve
C. Revere
D. Negate
E. Curtail

14. Indictment is most nearly opposite in meaning to

A. Crime
B. Tirade
C. Harangue
D. Punishment
E. Eulogy

15. Ponderous is most nearly opposite in meaning to

A. Abiding
B. Deceased
C. Dull
D. Sprightly
E. Serpentine

Questions 16-20 are sentence completions. Choose the answer which contains the words that best
fit the blanks and complete the meaning of the given sentence.

16. We appreciated his ____ summary of the situation; he wasted no words yet put his point most
persuasively.

A. trite
B. succinct
C. timorous
D. ponderous
E. fractious

17. Waste management is a ____ problem for the modern city, a problem that is never likely to
be addressed unless we change our ____ attitude to recycling.
A. growing - flexible
B. minor - positive
C. trivial - ambivalent
D. perennial - lax
E. recurring - stringent

18. Our accountant is most ____; he has never made a mistake in all the years he has worked for
us.

A. curmudgeonly
B. cantankerous
C. morose
D. meticulous
E. lethargic

19. When he was described on stage as a ____, Johnson threatened to sue the comedian for ____,
claiming that he was as dexterous as the next man.

A. lummox - slander
B. conjuror - libel
C. wizard - misrepresentation
D. failure - compensation
E. criminal - embezzlement

20. The journalist ____ the efforts of the drug squad to control drug pedaling, claiming that the
police team had actually made the problem worse.

A. disparaged
B. described
C. desecrated
D. commended
E. misconstrued

Synonym Sets test results


Practice Test 01
Question Your Answer Correct Answer Result

1. A A Correct

2. A C Wrong

3. A D Wrong

4. A B Wrong

5. A E Wrong

6. A A Correct

7. A C Wrong

8. A D Wrong

9. A B Wrong

10. A E Wrong

11. A A Correct

12. A B Wrong

13. A C Wrong

14. A E Wrong

15. A D Wrong

16. A B Wrong

17. A D Wrong

18. A D Wrong

19. A A Correct

20. A A Correct

Your score: 5 out of a possible 20


Word Focus Synonym Sets test 02
Questions 1-15 are Antonyms. Find the word most nearly OPPOSITE in meaning to the given
word.

1. Subservient is most nearly opposite in meaning to

A. Raucous
B. Onerous
C. Commanding
D. Illustrious
E. Destitute

2. Morose is most nearly opposite in meaning to

A. Peripheral
B. Parochial
C. Equivocal
D. Sanguine
E. Sane

3. Intrepid is most nearly opposite in meaning to

A. Cowardly
B. Sedentary
C. Unprepossessing
D. Jaundiced
E. Wanton

4. Zealous is most nearly opposite in meaning to

A. Motley
B. Irrevocable
C. Nonchalant
D. Trenchant
E. Prudent

5. Estrange is most nearly opposite in meaning to

A. Make calm
B. Grow close
C. Turn away
D. Be normal
E. Go back

6. Vainglorious is most nearly opposite in meaning to

A. Introverted
B. Sardonic
C. Ostentatious
D. Humble
E. Consummate

7. Fortitude is most nearly opposite in meaning to

A. Dynamism
B. Valor
C. Sagacity
D. Penetration
E. Pusillanimity

8. Reckless is most nearly opposite in meaning to

A. Circumspect
B. Meteoric
C. Accountable
D. Insufficient
E. Pertinent
9. Propensity is most nearly opposite in meaning to

A. Poverty
B. Aversion
C. Ambiguity
D. Embodiment
E. Censure

10. Lassitude is most nearly opposite in meaning to

A. Pulchritude
B. Inanity
C. Turbidity
D. Clemency
E. Vigor

11. Stalwart is most nearly opposite in meaning to

A. Prolific
B. Generous
C. Fickle
D. Fortunate
E. Grand

12. Scintillating is most nearly opposite in meaning to

A. Florid
B. Equitable
C. Exhaustive
D. Dull
E. Polished

13. Melancholy is most nearly opposite in meaning to


A. Tuneful
B. Sanguine
C. Apocryphal
D. Stunted
E. Disparate

14. Affected is most nearly opposite in meaning to

A. Hygienic
B. Mannered
C. Happy
D. Pretty
E. Simple

15. Wary is most nearly opposite in meaning to

A. Uncritical
B. Worried
C. Lively
D. Impeccable
E. Salubrious

Questions 16-20 are sentence completions. Choose the answer which contains the words that best
fit the blanks and complete the meaning of the given sentence.

16. Susie had been offended when the director ____ her ideas on set design, but she walked into
the meeting with a ____ air that belied the damage to her self-esteem.

A. repudiated - nonchalant
B. challenged - disappointed
C. mocked - morose
D. lauded - carefree
E. amended - aggressive

17. A naturally ____ person, my grandmother refused to succumb to our ____ mood and soon
had us laughing and joking again.
A. vivacious - obstreperous
B. warm - jovial
C. diffident - circumspect
D. effervescent - somber
E. dour - dejected

18. With a natural ____ languages, Brunt ____ German much more rapidly than did his less
linguistically gifted brother.

A. apathy towards - spoke


B. bent for - assimilated
C. dislike of - learned
D. inclination towards - tired of
E. aversion to - gave up on

19. Some ____ vegetarians ____ meat-eating not for health reasons, but on moral grounds: they
passionately believe that it is immoral to contribute to the suffering of animals.

A. life-long - tolerate
B. committed - accept
C. fervent - eschew
D. fanatic - condone
E. lapsed - recommend

20. The King, surprised that someone normally so ____ had suddenly found the courage to
question state policy, responded to the courtier’s ____ by having him imprisoned.

A. querulous - prevarication
B. pusillanimous - audacity
C. condescending - insolence
D. lethargic - vacillation
E. slavish - treason

Synonym Sets test results


Practice Test 02
Question Your Answer Correct Answer Result

1. A C Wrong

2. A D Wrong

3. A A Correct

4. A C Wrong

5. A B Wrong

6. A D Wrong

7. A E Wrong

8. A A Correct

9. A B Wrong

10. A E Wrong

11. A C Wrong

12. A D Wrong

13. A B Wrong

14. A E Wrong

15. A A Correct

16. A A Correct

17. A D Wrong

18. A B Wrong

19. A C Wrong

20. A B Wrong

Your score: 4 out of a possible 20


Word Focus Word Roots test 01
Questions 1-15 are Antonyms. Find the word most nearly OPPOSITE in meaning to the given
word.

1. Diurnal is most nearly opposite in meaning to

A. Eternal
B. Quotidian
C. Repetitive
D. Nocturnal
E. Ephemeral

2. Cosmopolitan is most nearly opposite in meaning to

A. Worldly
B. Respectable
C. Parochial
D. Weighty
E. Unfriendly

3. Agnostic is most nearly opposite in meaning to

A. Believer
B. Philosopher
C. Procrastinator
D. Skeptic
E. Leader

4. Omnipotent is most nearly opposite in meaning to

A. Poor
B. Febrile
C. Monolithic
D. Puissant
E. Powerless

5. Vociferous is most nearly opposite in meaning to

A. Aggressive
B. Meek
C. Ebullient
D. Sonorous
E. Thoughtful

6. Monochrome is most nearly opposite in meaning to

A. Boring
B. Variegated
C. Multidimensional
D. Complex
E. Bicameral

7. Mollify is most nearly opposite in meaning to

A. Ennoble
B. Decorate
C. Placate
D. Rile
E. Erode

8. Chronic is most nearly opposite in meaning to

A. Repeated
B. Enigmatic
C. Acute
D. Harmless
E. Unperturbed
9. Tremulous is most nearly opposite in meaning to

A. Firm
B. Threadbare
C. Familiar
D. Fragile
E. Dissembling

10. Equivocation is most nearly opposite in meaning to

A. Dissonance
B. Dispersal
C. Inequity
D. Mendacity
E. Directness

11. Heterogeneous is most nearly opposite in meaning to

A. Uniform
B. Homiletic
C. Diverse
D. Morose
E. Centered

12. Erratic is most nearly opposite in meaning to

A. Vibrant
B. Virtuous
C. Endless
D. Discrete
E. Constant

13. Amble is most nearly opposite in meaning to


A. Meander
B. Dilate
C. Deviate
D. Swerve
E. Scurry

14. Irascible is most nearly opposite in meaning to

A. Happy
B. Jaundiced
C. Jovial
D. Even-tempered
E. Permanent

15. Inerrant is most nearly opposite in meaning to

A. Fallible
B. Outspoken
C. Vibrant
D. Convoluted
E. Indelible

Questions 16-30 are Synonyms. Find the word most SIMILAR in meaning to the given word.

16. Provocative is most similar in meaning to

A. Cautious
B. Neutral
C. Inflammatory
D. Scandalous
E. Apocryphal

17. Emollient is most similar in meaning to


A. Compressing
B. Soothing
C. Allergenic
D. Controlling
E. Arousing

18. Amiable is most similar in meaning to

A. Affable
B. Inflexible
C. Overwrought
D. Abiding
E. Unfriendly

19. Circumspect is most similar in meaning to

A. Narrow-minded
B. Lenient
C. Cavalier
D. Prudent
E. Tight-fisted

20. Loquacious is most similar in meaning to

A. Demented
B. Covetous
C. Talkative
D. Lucid
E. Arrogant

21. Credulous is most similar in meaning to

A. Disreputable
B. Impecunious
C. Believable
D. Praiseworthy
E. Gullible

22. Cogitate is most similar in meaning to

A. Blunder
B. Dream
C. Dissemble
D. Regurgitate
E. Ponder

23. Pugnacious is most similar in meaning to

A. Supine
B. Aggressive
C. Lively
D. Rapacious
E. Vulpine

24. Circumvent is most similar in meaning to

A. Delineate
B. Avoid
C. Increase
D. Allow
E. Stop

25. Placid is most similar in meaning to

A. Vapid
B. Shiny
C. Noisy
D. Calm
E. Transparent
26. Intrepid is most similar in meaning to

A. Alert
B. Stupid
C. Valorous
D. Unwary
E. Wasteful

27. Ambivalent is most similar in meaning to

A. Cheerful
B. Even-tempered
C. Obese
D. Bilingual
E. Indecisive

28. Aberration is most similar in meaning to

A. Deviation
B. Intonation
C. Elevation
D. Alienation
E. Embarkation

29. Complacent is most similar in meaning to

A. Intolerant
B. Docile
C. Erudite
D. Smug
E. Calm

30. Homogeneous is most similar in meaning to


A. Sparse
B. Elegant
C. Unafraid
D. Unwavering
E. Standardized

Questions 31-40 are sentence completion questions. Choose the answer which contains the
words that best fit the blanks and complete the meaning of the given sentence.

31. Bancroft, who had inadvertently ____ his friend, wrote an ____ letter with the intention of
pouring oil over troubled waters.

A. alienated - amicable
B. humored - argumentative
C. harmed - asinine
D. offended - affected
E. mollified - extensive

32. It was with considerable ____ that my mother revealed the results of her ____ studies to my
grandfather as she had learned that my great- grandfather had been notorious criminal, a fact that
grandfather had been at pains to hide.

A. pride - hereditary
B. anxiety - anthropological
C. trepidation - genealogical
D. excitement - philosophical
E. annoyance - investigative

33. The old shepherd had a reputation for being ____ , and it came as no surprise to us that he
had a ____ attitude to us strangers, shouting and waving his fists to warn us off his land.

A. incognito - pugnacious
B. xenophobic - equivocal
C. philosophic - bellicose
D. philanthropic - benign
E. misanthropic - belligerent
34. As a child my grandfather used to refer to me as his little ____ because I suffered from ____ ,
and if my parents shut my bedroom door at night I would have nightmares and wander round the
house.

A. anachronism - terrors
B. progenitor - trepidation
C. wanderer - arachnophobia
D. somnambulist - claustrophobia
E. angel - acrophobia

35. The new teacher refused to take charge of the almost defunct Poetry Club, saying that it
would be too much trouble to ____ this ____ society.

A. circumscribe - vast
B. control - wayward
C. direct - defeated
D. resurrect - moribund
E. resuscitate - immortal

36. The speaker at the Marketing Convention resorted to frequent ____, yet these touches of
local color added a charm to his speech, and the audience went away with the impression that
they had heard one of the most ____ businessmen of the age.

A. evocations - fanciful
B. colloquialisms - eloquent
C. aberrations - vociferous
D. epigrams - omniscient
E. soliloquies - monotonous

37. The philosopher argues that far from being dead, ____ is a feature of human behavior in
modern times: his ____ lists articles containing many instances of people sacrificing themselves
for the good of others.

A. cognition - cosmology
B. altruism - bibliography
C. pugilism - appendix
D. parricide - epigram
E. philanthropy - epigraph

38. The device of the ____ narrator allows the author to write as though he is god: he sees all and
understands all.

A. omniscient
B. first person
C. imaginative
D. implacable
E. immortal

39. Even though he is now ____, my uncle continues to give the doctors cause for concern: they
say the ____ is not good and, in all probability, uncle will be confined to bed once again before
the week is out.

A. improving - chronology
B. optimistic - therapy
C. morbid - outlook
D. ambidextrous - result
E. ambulant - prognosis

40. For all their talk of ____ the land despoiled by mining, the local government has failed to
offer a ____ plan.

A. amortizing - feasible
B. clearing - credulous
C. regenerating - credible
D. reclaiming - quotidian
E. optimizing - monolithic

Word Roots test results


Practice Test 01
Question Your Answer Correct Answer Result
1. A D Wrong

2. A C Wrong

3. A A Correct

4. A E Wrong

5. A B Wrong

6. A B Wrong

7. A D Wrong

8. A C Wrong

9. A A Correct

10. A E Wrong

11. A A Correct

12. A E Wrong

13. A E Wrong

14. A D Wrong

15. A A Correct

16. A C Wrong

17. A B Wrong

18. A A Correct

19. A D Wrong

20. A C Wrong

21. A E Wrong

22. A E Wrong

23. A B Wrong

24. A B Wrong

25. A D Wrong
26. A C Wrong

27. A E Wrong

28. A A Correct

29. A D Wrong

30. A E Wrong

31. A A Correct

32. A C Wrong

33. A E Wrong

34. A D Wrong

35. A D Wrong

36. A B Wrong

37. A B Wrong

38. A A Correct

39. A E Wrong

40. A C Wrong

Your score: 8 out of a possible 40

Das könnte Ihnen auch gefallen